addthis

feed twitter facebook

1.2.08

Pathology Tricky Questions asked in saudi medical council examination

Autopsy of a 70-year-old woman reveals a papillary growth within the left ventricular chamber. The growth
consists of a small mass of finger-like projections attached to the mitral valve, without associated valvular or other
cardiac abnormalities. Histologically, each papillary structure is composed of a core of fibrous tissue lined by
thickened endothelium. The patient did not have any history of cardiac disease or evidence of thromboembolism.
Which of the following is the most likely diagnosis?


A. Acute mural thrombus

B. Cardiac myxoma

C. Infective endocarditis

D. Nonbacterial thrombotic endocarditis

E. Papillary fibroelastoma


Explanation:

The correct answer is E. Gross and microscopic features of this small mass in the left ventricle are entirely
consistent with papillary fibroelastoma. This lesion is not neoplastic, despite the sound of the name. It probably
results from organized thrombi forming on the endocardial surfaces of the mitral valve or left ventricular cavity.
Papillary fibroelastomas are usually clinically silent and are discovered at autopsy as an incidental finding.

An acute mural thrombus (choice A) would not have a core of fibrous tissue. Acute mural thrombosis usually
develops as a result of stasis in the ventricular cavities, in association with ventricular enlargement, myocardial
infarction, or ventricular aneurysm, for example. Thrombosis often develops in the atria when there is atrial
fibrillation.

Cardiac myxoma (choice B) is the most frequent primary cardiac neoplasm. It is benign and consists of stellate
mesenchymal cells within a myxoid background. Since the left atrium is the most frequent location, this tumor
can produce mitral stenosis by a ball-valve effect.

Both forms of endocarditis are associated with formation of vegetations attached to the surface of the
atrioventricular valves. Vegetations of infective endocarditis (choice C) are bulky and composed of fibrin,
bacteria, and inflammatory cells. Since nonbacterial thrombotic endocarditis (choice D) is caused by
hypercoagulable states, the vegetations consist of aggregates of fibrin but few inflammatory cells and no
bacteria.

Note that all of the above conditions may lead to systemic embolization. Fragments of vegetations, thrombi,
myxoma, and papillary fibroelastoma may detach and be released into the bloodstream, causing infarcts.



A 45-year-old man presents to his physician with hematuria. Renal biopsy demonstrates a focal necrotizing
glomerulitis with crescent formation. The patient has a history of intermittent hemoptysis and intermittent chest
pain of moderate intensity. A previous chest x-ray had demonstrated multiple opacities, some of which were
cavitated. The patient also has chronic cold-like nasal symptoms. Which of the following is the most likely
diagnosis?


A. Aspergillosis

B. Polyarteritis nodosa

C. Renal carcinoma metastatic to the lung

D. Tuberculosis

E. Wegener's granulomatosis


Explanation:

The correct answer is E. While in real life, other diseases (or combinations of diseases) may occasionally cause
concurrent pulmonary, sinus, and renal involvement, if you see this pattern on a test question, you should
immediately think of Wegener's granulomatosis. This is a rare focal necrotizing vasculitis of still unclear etiology,
which also features prominent granulomas, some of which are centered on the vascular lesions. The vasculitis
and granulomas can involve the entire respiratory tract, and an easily obtained nasal biopsy may sometimes
yield the diagnosis. The renal involvement is usually in the form of a necrotizing glomerulonephritis. The
disease typically affects middle aged men, and its formerly poor prognosis has been improved by corticosteroid
and cyclophosphamide therapy.

Aspergillosis (choice A) can cause prominent lung disease, but does not usually have renal involvement.

Polyarteritis nodosa (choice B) is a possibility, but usually spares the lungs.

Renal cell carcinoma (choice C) might cause lung nodules, but there is no evidence of tumor in the kidney.

Tuberculosis (choice D) can cause prominent lung disease, but does not usually have renal involvement.

A 55-year-old man presents to a physician with jaundice. Ultrasonography demonstrates a 5 cm mass in the head
of the pancreas. Endoscopic retrograde cholangiopancreatography with cytologic sampling demonstrates cells
with large hyperchromatic nuclei and a high nuclear/cytoplasmic ratio. A few small glands composed of these cells
are also seen in the cytologic preparation. The overall prognosis for this man will be most similar to that of a
patient with which of the following malignancies?


A. Adenocarcinoma of the breast

B. Adenocarcinoma of the colon

C. Adenocarcinoma of the esophagus

D. Adenocarcinoma of the prostate

E. Primary gastric lymphoma


Explanation:

The correct answer is C. The patient probably has pancreatic adenocarcinoma. This cancer carries one of the
worst prognoses, with a 3.5% overall 5 year survival rate despite all attempts at aggressive management. The
prognosis is also bleak with adenocarcinoma of the esophagus, with a 10% overall 5 year survival rate.

Adenocarcinoma of the breast (choice A) now has an overall 5 year survival rate of 60-70%.

Adenocarcinoma of the colon (choice B) now has an overall 5 year survival rate of 50-60%.

Adenocarcinoma of the prostate (choice D) now has an overall 5 year survival rate of 50-70%.

Primary gastric lymphoma (choice E) has an overall survival 5 year survival rate of 75-85%.





Biopsy of a persistent exophytic area on the vulva of a 60-year-old woman demonstrates a squamous epithelial
lesion. No koilocytes are seen. The lesions show papillary projections composed of disordered, squamous
epithelium with well-differentiated cells. The basement membrane at the dermal-epidermal junction is focally
disrupted by squamous cell groups extending deep into the dermis. Which of the following diagnoses is most
accurate?


A. Condyloma acuminatum

B. Extramammary Paget's disease

C. Vulvar intraepithelial neoplasia

D. Vulvar melanoma

E. Vulvar squamous cell carcinoma


Explanation:

The correct answer is E. The disruption of the basement membrane with cell groups extending deep into the
dermis indicates that this is invasive squamous cell carcinoma of the vulva, which may arise in vulvar
intraepithelial neoplasia, in condyloma, or spontaneously. The latter type (which is likely here) tends to be well
differentiated, while the former two are often poorly differentiated.

Condyloma acuminatum (choice A) would contain koilocytes and would not cross the basement membrane.

Extramammary Paget's disease (choice B) would probably contain individual tumor cells that stain for mucin.

The lesion of vulvar intraepithelial neoplasia (choice C) does not cross the basement membrane.

Vulvar melanoma (choice D) is composed of melanocytes that would mark with S100 or HMB-45.



Which of the following conditions is associated with overexpression of bcl-2?


A. Acute lymphoblastic leukemia

B. Burkitt lymphoma

C. Follicular lymphoma

D. Multiple myeloma

E. Small lymphocytic lymphoma


Explanation:

The correct answer is C.bcl-2 inhibits apoptosis; a (14;18) translocation resulting in overexpression of the bcl-2
protein in B lymphocytes causes apoptosis of neoplastic cells to be permanently inhibited, producing follicular
lymphoma. Follicular lymphoma involves the lymph nodes and bone marrow of older people and is
characterized by slow, but relentless, growth. Thus, this lymphoma allows a long survival even without any
treatment. Indeed, follicular lymphoma is resistant to aggressive therapy because of its low mitotic activity.

Acute lymphoblastic leukemia (ALL) (choice A) is a neoplasm of pre-B cells that affects children. Different
molecular mechanisms probably account for different cases. Hyperdiploidy, the Philadelphia chromosome,
t(12;21), and t(4;11) are variably found in patients with ALL.

Burkitt lymphoma (choice B) occurs in different clinical settings, including the African endemic, sporadic
nonendemic, and AIDS-associated forms. Histologically, all these forms are identical and are derived from
neoplastic transformation of B cells expressing surface IgM. Common to all forms are translocations involving
the c-myc oncogene (chromosome 8) and Ig loci (chromosome 14). These translocations lead to
overexpression of c-myc, which encodes a nuclear transcription protein, resulting in sustained cell replication.
The African form is associated with Epstein-Barr virus infection.

Multiple myeloma (choice D) is a neoplasm of terminally differentiated B lymphocytes that secrete large
amounts of monoclonal Ig or its fragments (light or heavy chains). Approximately 25% of cases have
translocations that lead to overexpression of FGFR3, a receptor for fibroblast growth factor implicated in control
of cell proliferation.

Small lymphocytic lymphoma (choice E), a B cell neoplasm, is equivalent to chronic lymphocytic leukemia.
Neoplastic cells resemble a normal subset of circulating mature B cells that express CD5. Various types of
chromosomal abnormalities have been found in small lymphocytic lymphoma/chronic lymphocytic leukemia, but
none affecting the bcl-2 locus.



A 10-year-old boy with history of epilepsy and mental retardation is brought to a specialty clinic for evaluation.
Physical examination is remarkable for several ovoid hypopigmented areas on the trunk and large numbers of red
and yellow papules on the face, particularly near the mouth. Biopsy of the papules demonstrates angiofibromata.
This patient is most likely to have which of the following central nervous system pathologies?


A. Acoustic neuromas

B. Capillary hemangioblastomas

C. Herniation of cerebellar tonsils into foramen magnum

D. Large cortical hamartomas

E. Leptomeningeal angiomatosis


Explanation:

The correct answer is D. The disease is tuberous sclerosis. The facial angiofibromata are also called adenoma
sebaceum, and the hypopigmented patches on the trunk are called ash-leaf spots. This disease is one of the
neurocutaneous disorders called phacomatoses. Tuberous sclerosis is inherited as an autosomal dominant
trait, and epilepsy and mental retardation are commonly seen in this disorder. Large, firm, white hamartomatous
nodules (tubers) are seen in the cortex and in subependymal sites. The tubers consist of aberrantly arranged
neurons and/or glia. Patients may also have pancreatic cysts, renal angiomyolipomas, and cardiac
rhabdomyomas. Rarely, an astrocytoma will arise in a tuber.

Acoustic neuromas (choice A) are a feature of neurofibromatosis Type II.

Capillary hemangioblastomas (choice B) are a feature of Von Hippel-Lindau syndrome.

Herniation of cerebellar tonsils (choice C) is a feature of Arnold-Chiari malformation.

Leptomeningeal angiomatosis (choice E) is a feature of Sturge-Weber disease.



A 30-year-old African American woman has a chest x-ray that shows hilar lymphadenopathy and diffuse
abnormalities of the lung parenchyma. Biopsy reveals non-caseating granulomas. Acid fast, silver, and PAS
stains on the granulomas are negative. Which of the following is the most likely diagnosis?


A. Coccidioidomycosis

B. Histoplasmosis

C. Leprosy

D. Sarcoidosis

E. Tuberculosis


Explanation:

The correct answer is D. Sarcoidosis is a common granulomatous disease, the etiology of which remains
unclear. It is a diagnosis of exclusion, and mycobacterial (acid fast) and fungal (silver stain, PAS) are used to
rule out infectious etiologies. The granulomas are characteristically small, non-caseating (hard) granulomas that
may contain giant cells. Sarcoidosis has an initial predilection for the lungs and the hilar lymph nodes, but with
time may spread to involve many organs. In the United States, sarcoidosis is much more common in people of
African ancestry. In Europe, it mainly affects whites. There is a slight female preponderance. Patients can have
a progressive course or experience exacerbations and remissions. Some patients recover with little residual
damage; those who die typically do so from pulmonary fibrosis and cor pulmonale.

The causative agent of coccidioidomycosis (choice A), Coccidioides immitis, or the causative agent of
histoplasmosis (choice B), Histoplasma capsulatum, would stain with PAS and silver stain.

The causative agent of leprosy (choice C), Mycobacterium leprae, or the causative agent of tuberculosis
(choice E), Mycobacterium tuberculosis, would stain with acid fast stain.



A patient presents with a severe form of atopic asthma. Which of the following changes would most likely be
found in this patient's blood?


A. Basophilic leukocytosis

B. Eosinophilic leukocytosis

C. Lymphocytosis

D. Monocytosis

E. Neutrophilic leukocytosis


Explanation:

The correct answer is B. An increased number of eosinophils (AKA eosinophilia) occurs in association with
several conditions, the most frequent of which are immune-mediated diseases (e.g., asthma, hay fever, and
pemphigus vulgaris) and parasitic infestations. This is due to an absolute increase in the number of circulating
eosinophils, brought about by IL-5, which stimulates differentiation of eosinophilic precursor cells in the bone
marrow.

Basophilic leukocytosis (choice A) is a rare event that is sometimes observed in association with chronic
myelogenous leukemia. It is not seen in patients with asthma.

Lymphocytosis (choice C) may result from a vast array of conditions, but it is not typical of asthma or other
allergic diseases. Lymphocytosis may develop in response to a number of infections (e.g., brucellosis,
whooping cough, hepatitis, infectious mononucleosis, and tuberculosis) or manifest as part of chronic
lymphocytic leukemia.

Monocytosis (choice D) refers to an increase in number of monocytes, which are circulating macrophages.
Chronic infections (e.g., tuberculosis, rickettsiosis, and malaria) and chronic inflammatory conditions (e.g.,
collagen vascular diseases and inflammatory bowel disease) are the most common underlying causes.

Neutrophilic leukocytosis (choice E) is a typical systemic reaction to acute and chronic infections, especially
those due to bacteria. The increase in neutrophil number is mediated by IL-1 and TNF, which induce a rapid
release of neutrophils from the bone marrow in acute infections and stimulate proliferation of bone marrow
precursors in chronic infections.



A patient with long term severe hypertension develops progressive dementia. CT scan of the head demonstrates
a diffuse loss of deep hemispheric white matter. Which of the following terms best describes the pathological
process that is occurring?


A. Anemic infarcts

B. Hemorrhagic infarcts

C. Hypertensive encephalopathy

D. Lacunae

E. Subcortical leukoencephalopathy


Explanation:

The correct answer is E. This patient has subcortical leukoencephalopathy (Binswanger's disease), which is one
of the neurologic syndromes associated with hypertension. It is uncommon, but obviously devastating. The
histologic findings are diffuse, irregular loss of axons and myelin accompanied by widespread gliosis. Small
infarcts may be seen in the frontal lobes. The pathologic mechanism may be damage caused by severe
arteriolosclerosis. None of the other choices would produce diffuse subcortical white matter involvement.

Anemic infarcts (choice A) can be seen in hypertensive patients as a consequence of atherosclerotic
thromboembolic events.

Hemorrhagic infarcts (choice B) can also be seen in hypertensive patients as a consequence of atherosclerotic
thromboembolic events followed by reperfusion. They tend to occur in gray matter or at the gray-white junction.

Hypertensive encephalopathy (choice C) is an acute generalized dysfunction of the brain that can occur in
malignant hypertension or other very severe hypertensive processes. The primary pathology is seen in cerebral
vessels, although cerebral edema may be present.

Lacunae (choice D) are small necrotic foci in deep gray matter (especially basal ganglia and thalamus) seen in
some hypertensive patients.



What percentage of Down's syndrome patients also have congenital cardiovascular disease?


A. 0.1%

B. 3%

C. 20%

D. 50%

E. 90%


Explanation:

The correct answer is C. This fact is worth remembering: one-fifth of Down's syndrome patients have
congenital cardiovascular disease, most commonly an ostium primum type of ASD and/or a ventricular septal
defect. This is a sufficiently high incidence to justify at least one echocardiogram in each of these children's
lives. Affected children are also particularly vulnerable to seizures, and as adults may develop an
Alzheimer-like dementia in their mid 40s.



A 50-year-old man presents with renal colic and an intravenous pyelogram demonstrates "clumps" of contrast
medium limited to the medulla. Multiple small stones are also seen. Blood chemistries are all within normal limits.
What is the most likely explanation for these findings?


A. Adult polycystic renal disease

B. Horseshoe kidney

C. Infantile polycystic renal disease

D. Medullary sponge kidney

E. Renal dysplasia


Explanation:

The correct answer is D. This is a classic presentation for medullary sponge kidney, which has a male
predominance and typically presents at 40-60 years of age. Histologically, medullary sponge kidney shows
multiple small cysts lined by columnar or cuboidal epithelium localized to the medullary collecting tubules. The
cysts can contain laminated concretions of calcium phosphates. Renal failure is rare in patients with medullary
sponge kidney and the pathogenesis for the lesion has not been clarified. The rare uremic medullary sponge
kidney is distinguished from the more common form by occurrence in 20-30 year-olds, salt-losing nephropathy,
and progression to renal failure.

The kidney of adult polycystic disease (choice A) is massively enlarged and filled throughout with round cysts
of varying sizes. Adult polycystic kidney disease often presents with hypertension rather than renal failure or
stones.

Horseshoe kidney (choice B) involves fusion of the upper or lower (most common) pole of the kidney. It is fairly
common (as high as 1:500), and is typically an incidental finding at autopsy.

Infantile polycystic kidney (choice C) produces a small kidney with round medullary cysts and "radiating" linear
cortical cysts.

Renal dysplasia (choice E) can also cause cystic change in a kidney, but typically involves only the medulla
and cortex of part of the kidney.



A 75-year-old female presents to the doctor with a chief complaint of vaginal spotting. She has been
post-menopausal for 25 years and does not take hormones. An ultrasound is performed, and shows a mass in
the uterine fundus. A hysterectomy is performed, and pathologic examination of the removed uterus reveals a
malignant tumor of the endometrial glands and stroma. Which of the following is the most likely diagnosis?


A. Endolymphatic stromal myosis

B. Endometrial carcinoma

C. Endometrial stromal sarcoma

D. Leiomyosarcoma

E. Malignant mixed müllerian tumor


Explanation:

The correct answer is E. Malignant mixed müllerian tumor is a tumor with 2 components, stromal and epithelial
(endometrial glands), both of which are malignant. This is a rare and highly aggressive tumor that has a 25%
5-year survival rate. It usually affects older patients and presents with post-menopausal bleeding. The stromal
component can contain metaplastic components such as cartilage and bone. Interestingly, usually only the
epithelial component metastasizes.

Endolymphatic stromal myosis (choice A) is a type of endometrial stromal tumor of intermediate malignancy. It
appears as small pieces of stroma between myometrial bundles that infiltrate lymph channels. Patients may
have pain or bleeding, or may be asymptomatic. Recurrences happen late in the course of the disease (years)
in half of patients and metastasis occurs in 15%. There is no epithelial component, so this is an incorrect
choice.

Endometrial carcinoma (choice B) is a malignancy of the epithelial glandular component of the endometrium.
Abnormal bleeding is the usual presentation. High estrogen states cause this tissue to proliferate. There is no
stromal component of this tumor, so this is an incorrect choice.

Endometrial stromal sarcoma (choice C) is a true sarcoma arising from the endometrial stroma that infiltrates
the myometrium and invades vessels. There is no epithelial component.

Leiomyosarcoma (choice D) is a true sarcoma arising from the uterine smooth muscle. It commonly has satellite
lesions within the uterus. Leiomyosarcomas usually recur after removal; survival is greater with well
differentiated lesions. Poorly differentiated lesions have a 10 to 15% 5-year survival rate. Distant metastasis is
via blood vessels. There is no epithelial component.



An 80-year-old man has low back pain. An x-ray of the lower back and pelvis shows sclerotic changes in the lower
vertebrae and in focal areas throughout the pelvis. The radiologist's report states that the sclerotic changes may
represent osteoarthritis; however, metastatic prostate cancer cannot be excluded. Which of the following is most
cost-effective in the initial work-up of this patient?


A. Bone marrow aspirate and biopsy

B. Digital rectal exam

C. Prostate-specific antigen

D. Radionuclide bone scan

E. Serum alkaline phosphatase


Explanation:

The correct answer is B. Osteoarthritis is the most common rheumatologic disease, the prevalence of which
increases with age. It primarily involves weight-bearing joints, hence its distribution in the lower vertebrae, pelvic
bones, and proximal femur. Sclerotic bone, representing reactive bone formation, develops as a reaction to
injury and is responsible for the slightly elevated serum alkaline phosphatase levels that normally occur in much
of the elderly population. If prostate cancer with osteoblastic (bone-forming) metastases to the vertebral column
and pelvis were present in this patient (stage D disease), a digital rectal exam would be the most cost-effective
initial step in the work-up. With advanced prostate cancer, the gland would very likely be enlarged and hard
("stony").

A bone marrow aspirate and biopsy (choice A) is not usually part of the normal work-up of possible metastatic
prostate cancer and has no place in the evaluation of osteoarthritis.

A prostate-specific antigen (PSA; choice C) level should be ordered in this patient, but not as the initial step in
the work-up, since it does not distinguish hyperplasia from cancer and is fairly expensive. In known cases of
prostate cancer, the PSA is a measure of tumor burden and is used to monitor recurrences when following
patients who have been treated for prostate cancer.

A radionuclide bone scan (choice D) is commonly used to rule out metastasis in patients with prostate cancer. It
is expensive and is not used as a screening test for prostate cancer.

The serum alkaline phosphatase (choice E) is typically elevated in metastatic prostate cancer due to
osteoblastic activity in the metastatic foci. However, an elevated serum alkaline phosphatase is non-specific,
since it may be slightly increased in osteoarthritis (reactive bone formation) as well as in liver disease.



A 40-year-old woman presents to the emergency department with severe abdominal pain localized to the right
upper quadrant. A urine sample is taken for rapid dipstick reagent strip analysis. A positive result for which of the
following substances would most strongly suggest gallstone disease as a possible cause of her abdominal pain?


A. Bilirubin

B. Glucose

C. Nitrite

D. Protein

E. Urobilinogen


Explanation:

The correct answer is A. A small gallstone passing into the common bile duct can cause obstructive jaundice
with conjugated hyperbilirubinemia. The conjugated bilirubin will spill into the urine, causing the bilirubin square
on the reagent strip to react. This strip may also react with other causes of intrahepatic or extrahepatic
obstructive jaundice, so it is not completely specific. It is worth becoming very familiar with the strengths and
weaknesses of reagent strip technology, however, since this information may be available hours before serum
chemistry values are reported.

High glucose (choice B) in urine suggests diabetes mellitus.

High nitrite (choice C) in urine suggests urinary tract infection.

High protein (choice D) in urine suggests renal disease or myeloma.

Urobilinogen levels (choice E) in obstructive jaundice can be normal, raised, or lowered; consequently, they are
not diagnostically helpful in the setting described in the question stem unless urinary bilirubin is negative.



A 60-year-old man presents to his physician after a routine screening test indicates hyperlipidemia. Physical
examination reveals raised, irregular, yellow papules in the skin of the soft tissues below the eyes. Biopsy of
these lesions would most likely show which of the following?


A. Benign nevus cells

B. Malignant nevus cells

C. Microscopic blisters

D. Munro microabscesses

E. Multinucleated giant cells


Explanation:

The correct answer is E. The lesions are xanthomas, which are tumor-like dermal collections of foamy
histiocytes containing cholesterol and lipids. The lesions may also contain multinucleated giant cells (Touton
giant cells) with clustered nuclei and foamy cytoplasm. Xanthomas may be idiopathic or may be related to
hyperlipidemia or lymphoproliferative malignancies (e.g., leukemias and lymphomas).

Nevus cells are a type of melanocyte. Benign nevus cells (choice A) are a feature of the common nevus, or
mole. Malignant nevus cells (choice B) are a feature of malignant melanoma.

Microscopic blisters (choice C) at the dermal/epidermal junction are a feature of dermatitis herpetiformis.

Munro microabscesses (choice D), small collections of neutrophils in the cornified epidermis, occur in psoriasis.



A 17-year-old male develops a painless, firm mass beneath the nipple of his left breast. The mass is mobile, and
no fluid can be expressed from the breast. The right breast is normal to examination. Which of the following
conditions does this mass most likely represent?


A. Fibrocystic changes

B. Gynecomastia

C. Intraductal papilloma

D. Invasive duct carcinoma

E. Invasive lobular carcinoma


Explanation:

The correct answer is B. The most common breast mass in men, especially under 25 years of age, is
gynecomastia-a benign proliferation of ductal and stromal elements of the breast. It is generally an idiopathic
condition, probably related to pubertal hormonal changes.

Fibrocystic changes (choice A) in the breast, which reflect physiological responses in the breast to cyclical
levels of sex hormones, are not observed in men. Fibrocystic changes are most common in the late
reproductive years and include fibrosis, cyst formation, and a variety of epithelial changes such as hyperplasia
and apocrine metaplasia.

Intraductal papillomas (choice C) are benign neoplasms commonly evolving in the major lactiferous ducts
beneath the nipple. They most commonly present with a bloody nipple discharge and are rare in men.

Carcinoma of the male breast (choices D and E) is rare, and almost always develops in the breasts of elderly
men. Male breast carcinomas have a somewhat worse prognosis than their female counterpart. Grossly and
microscopically they resemble ductal carcinoma in the female.



Physical examination of a 45-year-old man who looks much older than his stated age demonstrates thin arms and
legs, a swollen abdomen, red tongue, dry, thin, and slightly yellow skin, gynecomastia, testicular atrophy, multiple
spider angiomas, tremor, yellow discoloration of sclera, and short-term memory loss. Which of the following
conditions is most strongly suggested by these findings?


A. Bronchogenic carcinoma

B. Colon carcinoma

C. Congestive heart failure

D. Glomerulonephritis

E. Hepatic cirrhosis


Explanation:

The correct answer is E. The physical examination is typical for an advanced alcoholic with hepatic cirrhosis. It is
important to recognize these symptoms, as these patients are notorious for "underestimating" and even denying
their alcohol use.

Bronchogenic carcinoma (choice A) typically presents with cough and/or respiratory changes, but can present
with mass effects in the chest or involvement of mediastinal nerves or vessels.

Colon cancer (choice B) typically presents with changes in the stool or bowel habits.

Congestive heart failure (choice C) is typically heralded by shortness of breath or peripheral edema, or both.

Glomerulonephritis (choice D) typically presents with changes in quality or quantity of urine, and, possibly, fluid
retention.



A 54-year-old white male presents with gradual onset of mild dementia, ataxic gait, and startle myoclonus. An MRI
scan is normal, and an examination of his cerebrospinal fluid reveals no abnormalities, but the patient's EEG is
remarkable for recurrent bursts of high-voltage slow waves. Over the next 6 months, the patient's dementia
rapidly worsens, accompanied by general hypertonicity and profound dysarthria. The patient dies shortly
thereafter. Which of the following is the mostly likely neuropathological finding on autopsy?


A. Cerebellar hyperplasia

B. Diffuse spongiform change

C. Multiple lacunar infarcts

D. Negri bodies

E. Neurofibrillary tangles


Explanation:

The correct answer is B. The rapidly progressive dementia in this case is characteristic of Creutzfeldt-Jakob
disease (CJD). The dementia is usually accompanied by motor dysfunction and abnormal EEG activity, as
described in the question stem. The pathological hallmark of this disease is spongiform change in the gray
matter. Death usually occurs within 6-12 months of disease onset.

At autopsy, the cerebellum in CJD appears atrophic, not hyperplastic (choice A).

Multiple lacunar infarcts (choice C) are seen with vascular dementia, and patients typically present with focal
neurological signs. Additionally, vascular dementia typically presents with a more gradual decline in cognitive
function, measured in years rather than months.

Negri bodies (choice D) are pathognomic for rabies, which does not cause dementia.

Neurofibrillary tangles (choice E) are often seen in patients with Alzheimer's disease. Alzheimer's disease
primarily affects the higher order association cortex; motor dysfunction is not generally observed. Additionally,
in Alzheimer's disease, there is a more gradual decline in cognitive function, measured in years rather than
months.



A 38-year-old woman is in her first pregnancy, which has been uneventful until the 34th week, when she
develops swelling of feet and hands. An obstetric check-up reveals that she also has hypertension and
proteinuria. Laboratory analysis shows elevated aspartate aminotransferase (AST) and alanine aminotransferase
(ALT) and slightly decreased platelets. The initial event in the pathogenesis of her condition is thought to be
which of the following?


A. Chorioamnionitis

B. Disseminated intravascular coagulation

C. Maternal hypertension

D. Maternal renal ischemia

E. Placental ischemia


Explanation:

The correct answer is E. This patient's condition is a classic third trimester complication referred to as toxemia
of pregnancy, or preeclampsia. It occurs in 6% of all pregnancies but is more frequent in primiparas. Although
the pathogenesis is still unclear, the first event appears to be placental ischemia, probably due to abnormalities
in the trophoblast and alterations in the maturation of placental vessels. The trophoblast of invading placental
vessels fails to acquire the characteristics of normal endothelial cells, with subsequent alterations in blood flow.
Placental ischemia then triggers the release of thromboplastic substances, increases renin synthesis, and
reduces prostaglandin E levels.

Chorioamnionitis (choice A) is an infection of chorioamnionic membranes due to bacteria that ascend through
the vaginal canal. This is an important cause of spontaneous abortion in the second and third trimester, but it
plays no role in the pathogenesis of toxemia of pregnancy.

The release of thromboplastic substances may cause disseminated intravascular coagulation (DIC) (choice B).

Increased renin and reduced prostaglandin E mediate increased sensitivity to angiotensin, leading to maternal
hypertension (choice C).

Ischemic damage to maternal organs, including brain, liver, and kidneys (choice D), results from thrombotic
occlusion of arterioles and capillaries as a consequence of DIC.

Overall, the clinical picture of toxemia is due to DIC-mediated ischemic damage to brain (changes in mental
status and convulsions), liver (elevated liver enzymes), and kidneys (proteinuria, leading to peripheral edema).
A manifestation of toxemia is HELLP syndrome, which stands for hemolysis, elevated liver enzymes, and low
platelets.



Molecular studies on an abdominal lymph node containing lymphoma demonstrate t(2;8)(p12;q24) translocation.
This is most compatible with which of the following diseases?


A. Burkitt's lymphoma

B. Mantle cell lymphoma

C. Multiple myeloma

D. Small cell lymphoma

E. Small cleaved cell lymphoma


Explanation:

The correct answer is A. Burkitt's lymphoma is actually associated with three translocations. The common
variant t(8;14)(q24;q32), involving the oncogene myc on chromosome 8, and the heavy immunoglobulin chain
on chromosome 14. The other two variants are: t(8;22)(q24;q11), involving myc and the lambda light chain
immunoglobulin site, and t(2;8)(p12;q24), involving the kappa light chain and myc.

Mantle cell lymphoma (choice B), multiple myeloma (choice C), and small (not cleaved) cell lymphoma (choice
D) are associated with the t(11;14)(q13;q32) translocation involving bcl -1 and the heavy chain site.

Small cleaved cell lymphoma (choice E) is associated with t(14;18)(q 32;q21), involving the immunoglobulin
chain site and bcl-2.


A newborn infant is noted to have numerous, light brown macules dispersed across her skin. The significance of
this feature is due to its strong association with the development of which of the following tumors?


A. Basal cell carcinoma

B. Neuroblastoma

C. Neurofibroma

D. Retinoblastoma

E. Wilms' tumor


Explanation:

The correct answer is C. Congenital "cafe au lait" spots are present in more than 90% of patients with
neurofibromatosis. This autosomal dominant disorder is characterized by multiple neural tumors, especially
neurofibromas, pigmented hamartomas of the iris (Lisch nodules), and cafe au lait spots, which usually occur
over nerve trunks. Although the majority of neurofibromas in this disease are benign, the tumors can be quite
disfiguring and psychologically damaging.

Basal cell carcinoma (BCC; choice A) is a common, slow-growing tumor of sun-exposed skin. It develops in
adulthood after years of chronic sun damage. Although malignant melanoma may arise from large congenital
nevi, BCC does not.

Neuroblastoma (choice B) is a common childhood tumor that arises anywhere along the sympathetic chain, and
most commonly in the adrenal medulla. The tumor usually presents as an abdominal mass and is not
associated with skin findings.

The ocular neoplasm associated with neurofibromatosis is the Lisch nodule, not the retinoblastoma (choice D).
Retinoblastoma is a neuroepithelial tumor usually identified by funduscopic examination of a child with visual
changes.

Wilms' tumor (choice E) is a neoplasm of primitive renal blastema that may be associated with congenital
malformations of visceral organs, notably the adrenals and gonads, but not skin. It also presents as an
abdominal mass, and is highly survivable with modern treatment modalities.



A neonate develops bile-stained vomiting and progressive abdominal distention, and does not pass meconium
over the first two days of life. The anus is patent, and the bowel loops are palpable. Plain radiograph shows
bubbly meconium in the right lower quadrant. No localized areas of constriction or other abnormalities are noted.
Which of the following is most likely etiologically related to this infant's condition?


A. Cystic fibrosis

B. Hirschsprung's disease

C. Meckel's diverticulum

D. Omphalocele

E. Polycystic kidney disease


Explanation:

The correct answer is A. The baby has meconium ileus, which is a manifestation of cystic fibrosis due to the
abnormally viscid pancreatic secretions which "get stuck" in the small bowel. Meconium ileus can cause gut
perforation with peritonitis and intraperitoneal calcifications (that may be visible on plain film). Meconium ileus
complicated by intestinal perforation or formation of fistulas to the bladder or vagina must be treated surgically.
Medical treatments for uncomplicated meconium ileus are now available, which use enemas, mucolytic agents,
or pancreatic enzymes.

Hirschsprung's disease (choice B) is a cause of congenital constipation related to absence of ganglion cells in a
segment of bowel. The aganglionic bowel segment is narrowed because the lack of peristalsis keeps stool from
moving into the segment. The distal rectum is always involved, and the lesion may extend proximally as far as
the small intestine. The bowel proximal to the lesion is usually dilated.

A Meckel's diverticulum (choice C) can form due to the persistence of the vitelline duct, which connects the
developing gut to the yolk sac. They are classically located in the distal ileum within 30 cm of the ileocecal
valve, and may contain ectopic pancreatic tissue or gastric mucosa.

Omphalocele (choice D) is characterized by herniation of abdominal viscera through the abdominal wall near
the umbilicus.

In polycystic kidney disease (choice E), cysts of the liver and pancreas may occur; meconium ileus is not
associated with this disorder.


A 69-year-old woman is brought to the emergency room after falling off a step stool and fracturing her hip. Her
past medical history is significant for several bouts of pneumonia during the past year. Laboratory results indicate
a normal white blood cell count, but platelets are decreased, and the erythrocyte sedimentation rate (ESR) is
elevated. X-rays reveal multiple lytic bone lesions. Serum electrophoresis demonstrates an M-protein spike.
Which of the following is the most likely diagnosis?


A. Chronic lymphocytic leukemia

B. Monoclonal gammopathy of uncertain significance

C. Multiple myeloma (plasma cell myeloma)

D. Plasmacytoma

E. Waldenström's macroglobulinemia


Explanation:

The correct answer is C. The patient is suffering from multiple myeloma, a neoplastic proliferation of plasma
cells (or their precursors) found within the bone marrow. These malignant cells are responsible for the
production of excessive amounts of immunoglobulin (usually IgG or IgA), producing an M- protein spike, and
increasing the ESR. The decreased platelet count probably reflects infiltration of the bone marrow by myeloma
cells. It is not unusual for patients with malignant myeloma to have recurring bacterial infections, particularly
pneumococcal pneumonia, because the overall production of normal immunoglobulins of all isotypes is
decreased. Lytic bone lesions due to infiltration by myeloma cells may lead to pathological fractures. All of the
other disorders listed can be associated with an M-protein spike, but this is a nonspecific finding.

Chronic lymphocytic leukemia (CLL; choice A) is ruled out by the normal white count, since this type of leukemia
is generally associated with an absolute lymphocytosis.

Patients with monoclonal gammopathy of undetermined significance (choice B) are generally asymptomatic,
although they have a predisposition for subsequently developing myeloma, lymphoma, amyloidosis, or
Waldenström's macroglobulinemia.

Plasmacytoma (choice D) (solitary myeloma) is a rare, isolated plasma cell neoplasm in bone or soft tissues. If
the primary cancer is in bone, it is likely to disseminate; extraosseous tumors tend to remain localized. Since this
patient had multiple lytic bone lesions, multiple myeloma is the correct diagnosis.

Waldenström's macroglobulinemia (choice E) is a disorder involving neoplasms of lymphocytoid plasma cells
that produce monoclonal IgM. Hypergammaglobulinemia produces hyperviscosity of the blood which, along with
infiltration by tumor produce the characteristic signs and symptoms. Weakness, weight loss, bone pain,
hepatosplenomegaly, and lymphadenopathy occur commonly.




A 55-year-old woman complains to her physician that the skin of her armpits and groin "keeps getting darker and
darker." Physical examination demonstrates velvety brown and warty skin in the axilla and groin. Biopsy of these
lesions shows a variably hyperplastic epidermis with many sharp peaks and valleys. Aside from cosmetic
considerations, which of the following is the primary medical significance of these lesions?


A. They may be a sign of immunosuppression

B. They may be a sign of visceral carcinoma

C. They may be easily superinfected

D. They may be malignant

E. They may be premalignant


Explanation:

The correct answer is B. The lesions are acanthosis nigricans, which looks somewhat like a mole or wart, but is
actually due to epidermal hyperplasia. Acanthosis nigricans can be seen in obesity, diabetes, and in patients
with underlying cancers, often adenocarcinomas of the chest or abdomen.

The lesions are not characteristic of immunosuppression (choice A) and are not easily superinfected (choice C).
They are also neither malignant (choice D) nor premalignant (choice E).



A 47-year-old woman undergoes endometrial biopsy because she has had repeated episodes of irregular
spotting between periods lately. The biopsy shows strips of endometrium bearing long, narrow, coiled glands
lined by a single layer of columnar epithelium showing regular, uniform, small nuclei and clear apical vesicles.
Which of the following is the most likely diagnosis?


A. Atypical hyperplasia

B. Complex hyperplasia without atypia

C. Proliferative endometrium

D. Secretory endometrium

E. Simple hyperplasia


Explanation:

The correct answer is D. Endometrial biopsies are often performed to evaluate patients with menstrual bleeding
abnormalities, particularly in perimenopausal or postmenopausal patients. While the biopsies are done to rule
out hyperplasias or cancer, most of the specimens actually show only proliferative or secretory (as in this
patient) endometrium. Secretory endometrium has the features noted in the question stem.

Atypical hyperplasia (choice A) is characterized by complex glands lined by cells showing features of atypia,
such as cytomegaly, increased nuclear cytoplasmic ratio, prominent nucleoli, and increased mitotic index.

Complex hyperplasia without atypia (choice B) is characterized by complex, branching glands without cellular
atypia.

Proliferative endometrium (choice C) is characterized by smaller, noncoiled, glands lined with columnar
epithelium without apical (secretory) vesicles.

Simple (cystic) hyperplasia (choice E) is characterized by cystically dilated glands without cellular atypia.



Which of the following conditions is the most frequent cause of spontaneous abortion in the first trimester of
pregnancy?


A. Abruptio placentae

B. Chorioamnionitis

C. Chromosomal abnormalities

D. Placenta previa

E. Trauma


Explanation:

The correct answer is C. At least 10% to 15% of normally fertilized and implanted ova are lost in the first
trimester of pregnancy because of spontaneous abortion. Studies using immunoassay of human chorionic
gonadotropin (hCG) for early diagnosis of pregnancy suggest that the percentage of fertilized ova lost in the
first trimester might be even higher. The great majority of these cases are attributable to chromosomal
abnormalities. Chromosomal studies are not routinely performed in such cases, but they are recommended
when a malformed fetus has been identified or when habitual or recurrent abortions occur.

Abruptio placentae (choice A), a complication of the third trimester, occurs when the placenta detaches
prematurely from its implantation site. Retroplacental hemorrhage develops within the space between placenta
and uterine wall, leading to interruption or severe reduction in the blood supply to the fetus.

Chorioamnionitis (choice B), a complication of the second and third trimesters, results from ascending infections
through the vaginal canal. Infection of chorioamnionic membranes may lead to premature rupture of
membranes and abortion or premature labor.

Placenta previa (choice D) is a placenta implanted in the lower segment of the uterus. When dilatation of this
segment begins in late pregnancy, a placenta previa may cause severe bleeding and lead to premature labor.

Surprisingly, trauma (choice E) is a rare cause of spontaneous abortion.



A 65-year-old woman has a long-standing dementing disorder characterized by deterioration in personality,
neglect of personal hygiene, impaired judgment, and disinhibited behavior. MRI demonstrates severe cortical
atrophy limited to the frontal lobes and anterior two thirds of the temporal lobes, while the remaining cortex is
preserved. No evidence of recent or remote infarcts is found. Which of the following diagnoses is most consistent
with these pathologic and clinical features?


A. Alzheimer disease

B. Creutzfeldt-Jacob disease

C. Dementia with Lewy bodies

D. Frontotemporal dementia

E. Vascular dementia


Explanation:

The correct answer is D. Not all dementing disorders manifest with the same clinical features. Although there is
considerable overlap in clinical symptomatology among different types of dementias, making clinical diagnosis
somewhat problematic, there are classic presentations that allow identification of a specific form of dementia
with a high degree of confidence. In this case, the patient has symptoms due to frontal lobe damage, eg,
disinhibition, impaired judgment, and personality changes. Furthermore, MRI demonstrates a specific pattern of
cortical atrophy, restricted to the frontal lobes and anterior portion of the temporal lobes. This combination
points toward a group of dementias called frontotemporal dementia, the most frequent form of which is Pick
disease. Other forms of frontotemporal dementia are very infrequent. Remember: frontal symptoms in
conjunction with frontotemporal atrophy = frontotemporal dementia/Pick disease.

Alzheimer disease (choice A) is the most frequent form of dementia in industrialized countries. Although
symptoms due to frontal damage may be present in Alzheimer disease, they are usually associated with a more
generalized impairment of higher neurologic functions, eg, language, memory, and learned movements. In
addition, cortical atrophy in Alzheimer disease is widespread and not limited to the frontal and anterior temporal
lobes.

Creutzfeldt-Jacob disease (choice B) represents the prototype of prion diseases. Cortical atrophy is not a
prominent feature of Creutzfeldt-Jacob disease, which manifests with personality changes, memory loss, and
seizures, leading to death after a rapid clinical course (a few months to 1 year).

Dementia with Lewy bodies, also known as diffuse Lewy body disease (choice C), is one of the most common
forms of dementia in Western countries, possibly more common than vascular dementia. It is characterized by
widespread formation of Lewy bodies in the substantia nigra, limbic cortex, and subcortical nuclei, such as the
basal nucleus of Meynert. Extrapyramidal symptoms similar to Parkinson disease manifest in this form of
dementia as a result of degeneration of dopaminergic pathways.

Vascular dementia (choice E) is an umbrella term encompassing dementing conditions that arise from pathology
of large or small cerebral vessels. It manifests with memory loss associated with focal neurologic symptoms
depending on the location of damage. MRI would identify old or recent infarcts, as well as white matter disease.
Conditions associated with vascular dementia include the following:
Multi-infarct dementia, which is caused by multiple, scattered brain infarcts secondary to atherosclerosis of
large arteries of the circle of Willis and/or carotid arteries.
Binswanger disease, which involves rarefaction of cerebral white matter and is caused by hypertension-related
arteriolosclerosis.
Lacunar infarcts, which consist of small (< 1 cm) infarcts in the striatum and thalamus; this condition is related
to arteriolosclerosis.



Biopsy of a reasonably well-demarcated mass of the nasopharynx demonstrates a plasma cell proliferation.
Serum electrophoresis shows a small monoclonal IgG spike. Bone marrow evaluation fails to demonstrate plasma
cell proliferation and no lesions are seen on extensive skeletal x-rays. Which of the following is the most likely
diagnosis?


A. Heavy chain disease

B. Monoclonal gammopathy of undetermined significance

C. Multiple myeloma

D. Plasmacytoma

E. Waldenström's macroglobulinemia


Explanation:

The correct answer is D. Plasmacytoma (solitary myeloma) involving soft tissue (lungs, nasopharynx, nasal
sinuses) is a plasma cell proliferation resembling multiple myeloma but without significant metastatic potential. In
contrast, some plasmocytomas involving bone eventually (up to 10 to 20 years) develop into frank multiple
myeloma.

Heavy chain diseases (choice A) constitute a group of rare lymphoplasmacytic malignancies in which excessive
amounts of a defective heavy immunoglobulin chain are produced. They may take the form of gamma
heavy-chain disease (from IgG), alpha heavy-chain disease (from IgA) or mu heavy-chain disease (from IgM);
malignant cells are usually present in marrow in all of these conditions.

Monoclonal gammopathy of undetermined significance (choice B) is a disease of elderly patients with a
monoclonal spike on serum or urine electrophoresis, but no identifiable mass or bone marrow lesion; 20% of
these patients eventually develop one of the other diseases listed in the answer choices.

Multiple myeloma (choice C) is a malignancy derived from a single plasma cell clone with significant metastatic
potential. Multiple lytic bone lesions are usually seen.

Waldenström's macroglobulinemia (choice E) is a malignancy of lymphoplasmacytic cells that secrete IgM. In this
disorder, the bone marrow is diffusely rather than focally infiltrated by lymphocytes, plasma cells, and hybrid
forms.


A 65-year-old woman presents with a 30-lb weight loss and malaise. Cancer is suspected. Which of the following
is the correct list, starting with the most prevalent, of the three most common causes of cancer in women?


A. Breast, lung, colon and rectum

B. Breast, uterus, lung

C. Colon and rectum, lung, ovary

D. Lung, breast, ovary

E. Ovary, uterus, lung


Explanation:

The correct answer is A. The correct female incidence sequence is breast (32%), lung (13%), and colon and
rectum (13%). The two major causes of cancer death in women are lung (23%) and breast (18%). In men, the
incidence sequence is prostate (32%), lung (16%), and colon and rectum (12%). The two leading causes of
male cancer deaths are lung (33%) and prostate (13%).

The uterus and ovary (choices B, C, D, and E) are not among the three organs most frequently affected by
cancer in women.


A 50-year-old woman presents with a 5-year history of headaches, generalized tonic-clonic seizures, and bilateral
leg weakness. Skull films reveal hyperostosis of the calvarium. Biopsy of the responsible lesion shows a whorling
pattern of the cells. Which of the following is the most likely diagnosis?


A. Arachnoid cyst

B. Glioblastoma multiforme

C. Meningioma

D. Metastatic breast cancer

E. Oligodendroglioma


Explanation:

The correct answer is C. The most likely diagnosis is an intracranial meningioma. Meningiomas are
slow-growing, benign tumors comprising 15% of intracranial tumors; they are most common in the elderly. They
originate from either dura mater or arachnoid and are sharply demarcated from brain tissue. Meningiomas often
incite an osteoblastic reaction in the overlying cranial bones. Microscopically, the meningioma cells have a
tendency to encircle one another, forming whorls and psammoma bodies. Clinically, they present as mass
lesions; seizures may occur. The superior parasagittal surface of the frontal lobes is a favorite site of origin.
This can often produce leg weakness, since the leg motor fibers that pass down through the internal capsule
originate in parasagittal cortical regions. Treatment of meningiomas is usually surgical.

Arachnoid cysts (choice A) are formed by splitting of the arachnoid membrane; most arachnoid cysts arise near
the Sylvian fissure. They may present with mass effect, but would be unlikely to produce seizures, prominent
focal signs, or reactive hyperostosis.

Glioblastoma multiforme (choice B) is an aggressive malignant astrocytoma that would likely have killed the
patient long before 5 years had elapsed.

Metastatic breast cancer (choice D) would generally look different microscopically (the whorling cell pattern is
characteristic of meningioma). It would be unlikely for metastatic cancer to cause a reaction in the overlying
bone, or to be present long enough to cause symptoms for 5 years.

Oligodendrogliomas (choice E) are glial tumors that could produce the described clinical picture, but usually do
not cause hyperostosis of the calvarium or exhibit the characteristic whorling cell pattern microscopically.

When a histologic section is taken of an abscess, many of the observed neutrophils show a degenerative change
in which the nucleus has undergone fragmentation. This process is known as


A. caseous necrosis

B. coagulative necrosis

C. karyolysis

D. karyorrhexis

E. pyknosis


Explanation:

The correct answer is D. Karyorrhexis refers to a pattern of nuclear degradation in which a pyknotic or partially
pyknotic nucleus undergoes fragmentation followed by complete lysis. This pattern is common in the
neutrophils present in acute inflammation.

The type of necrosis seen in an abscess is liquefactive necrosis. Caseous necrosis (choice A) is seen in
tuberculosis and some other granulomatous diseases; coagulative necrosis (choice B) is seen following
infarctions of many organs (other than the brain).

Karyolysis (choice C) is also a degenerative change affecting nuclei. In this case, however, it is seen as a
decrease in nuclear basophilia, which is presumably the result of DNAse activity.

Pyknosis (choice E) is characterized by nuclear shrinking and basophilia, apparently as a result of DNA
condensation.




A 52-year-old woman has recently undergone a breast resection for carcinoma. Based on the statistics for breast
cancer incidence, which of the following types of carcinoma does this patient most likely have?


A. Colloid (mucinous)

B. Invasive ductal

C. Invasive lobular

D. Medullary

E. Metastatic bronchogenic


Explanation:

The correct answer is B. Invasive ductal carcinoma is the most likely candidate. Of the various types of breast
adenocarcinoma, invasive ductal carcinoma is by far the most common variant, accounting for approximately
75% to 80% of all invasive breast carcinomas. Invasive ductal carcinoma develops from epithelial cells of the
terminal duct. Histologically, it is composed of small, glandular, ductlike structures, lined by variably anaplastic
cells. The most common mode of presentation is a palpable mass in the breast. Its prognosis depends mostly
on staging (spread of cancer) rather than grading (degree of differentiation).

The colloid (mucinous) variant (choice A) is relatively rare (about 1% to 2%) and occurs more frequently in
older women. Histologically, this carcinoma is characterized by abundant mucin secretion. It is associated with a
better prognosis than the ductal type.

Invasive lobular carcinoma (choice C) is the second most frequent histologic type of breast adenocarcinoma,
accounting for approximately 10% of all cases. Its presumed cell of origin is the lobular cell. The most typical
histologic characteristic is the presence of cancer cells lined up in orderly rows ("single-file").

Medullary carcinoma (choice D) tends to occur in younger women and is associated with a slightly better
prognosis. Although a malignant tumor, medullary carcinoma is well circumscribed and surrounded by a florid
lymphoplasmacytic reaction. The name is due to its soft consistency.

Metastatic cancer may involve the breast like any other organ. Bronchogenic carcinoma (choice E) may also
spread to the breast by lymphatic route or by contiguity, but this would be less likely than primary breast
cancer.



A 55-year-old man is brought to his physician's office with a 3-month history of progressive mental deterioration
in the form of memory loss, mood changes, and errors in judgment. His gait is unsteady, and he requires
assistance to prevent falling. He has no history of seizures, head trauma, or incontinence. Computed tomography
(CT) scan and lumbar puncture are unremarkable. Physical examination reveals hypertonicity of all extremities,
bilateral equivocal plantar response, ataxic gait, and myoclonic jerks in the lower extremities. What is the
mechanism by which this infectious agent causes its pathology?


A. Amyloid deposition

B. Autoimmune destruction

C. Chronic inflammation

D. Embolization and infarction

E. Toxin production


Explanation:

The correct answer is A. This is the classic presentation of Creutzfeldt-Jacob disease (CJD). Although the
pathogenesis is incompletely understood, these patients develop extracellular deposition of abnormal fibrillar
proteins in the brain, ie, amyloid.

Autoimmune destruction (choice B) is not indicated because there is no immunologic response to the
deposition of these extracellular proteins; thus there is no chronic inflammation (choice C)

Although embolization and infarction (choice D) could complicate the presentation in the age group typically
afflicted with CJD, these processes are not believed to have any direct role in this pathology.

No toxin is produced (choice E) to account for the presentation in CJD.


A patient consults a dermatologist about a skin lesion on her neck. Examination reveals a 1-cm diameter, red,
scaly plaque with a rough texture and irregular margins. Biopsy demonstrates epidermal and dermal cells with
large, pleomorphic, hyperchromatic nuclei. Which of the following conditions would most likely predispose this
patient to the development of this lesion?


A. Actinic keratosis

B. Compound nevus

C. Dermal nevus

D. Junctional nevus

E. Melanoma


Explanation:

The correct answer is A. The lesion is a squamous cell carcinoma of the skin. Actinic keratosis, which is a
hyperplastic lesion of sun-damaged skin, predisposes for squamous cell carcinoma. Another predisposing
condition to remember is xeroderma pigmentosum, which predisposes for both squamous cell and basal cell
carcinomas of skin.

A nevus is a mole, containing characteristic cells called nevocellular cells. If the nevocellular cells are located at
the dermal-epidermal junction (junctional nevus, choice D), in the dermis (dermal nevus, choice C), or both
(compound nevus, choice B), they do not predispose for squamous cell carcinomas of the skin. Malignant
melanoma (choice E), however, can arise in pre-existing nevi.


A 24-year-old woman gives birth to an apparently normal infant. The neonate begins feeding well by the second
day, then at ten days, suddenly develops gastrointestinal obstruction. Which of the following is the most likely
cause of this presentation?


A. Adhesions

B. Congenital pyloric stenosis

C. Hirschsprung's disease

D. Intussusception

E. Volvulus


Explanation:

The correct answer is D. All of the conditions listed can cause gastrointestinal obstruction, but the clinical
presentation is most suggestive of intussusception. In intussusception, there is telescoping of one bowel
segment into another, more distal segment. The disorder is relatively common in infants and children due to the
poor support offered by their thin mesentery. Intussusception produces intestinal obstruction, and it may
produce bowel ischemia or infarction by trapping mesenteric vessels along with the affected segment. In some
cases, the intussusception may be reduced by diagnostic barium enema.

Adhesions (choice A) can cause bowel obstruction following surgery or inflammatory bowel diseases.

Congenital pyloric stenosis (choice B) typically presents as projectile vomiting in a 3-4 week old baby.

Hirschsprung's disease (choice C), caused by absence of ganglion cells in the distal bowel, is usually
diagnosed in the first few days of life when there is a failure to pass meconium.

Volvulus (choice E) is due to rotation of bowel segments. This is usually a disease of the elderly.




A middle-aged man with a long history of sexual activity slowly develops testicular enlargement. Needle biopsy
demonstrates the presence of obliterative endarteritis with perivascular cuffing of lymphocytes and plasma cells.
A diffuse interstitial inflammation with edema and prominent plasma cell infiltrate is also present. Which of the
following is the most likely diagnosis?


A. Gonorrhea

B. Mumps

C. Nonspecific orchitis

D. Syphilis

E. Tuberculosis


Explanation:

The correct answer is D. This is one presentation of syphilis, which can involve the testis and epididymis in both
the acquired and congenital types of the disease. Characteristically, the testis is usually involved before the
epididymis. Microscopically, the testis may show either gumma formation or the findings described in the
question stem.

Gonorrhea (choice A) usually causes an acute epididymitis and orchitis with prominent neutrophils.

Mumps orchitis (choice B) would usually be accompanied by parotitis.

Nonspecific orchitis (choice C) is characterized by prominent neutrophils.

Tuberculosis orchitis (choice E) is characterized by granulomas with acid-fast bacteria.




A 70-year-old woman dies in a nursing home after a heart attack. The time of onset of her clinical
symptomatology and the cause of death are uncertain; furthermore, the possibility of neglect is being
considered. Therefore, an autopsy investigation is arranged. The forensic pathologist discovers acute
thrombosis involving the posterior descending branch of the right coronary artery with resultant myocardial
infarction (MI) in the posterior third of the interventricular septum. Histologically, there is coagulation necrosis
with associated abundant neutrophilic infiltration. Histiocytes and lymphocytes are scanty. Which of the following
is the approximate period between the onset of pain (ie, beginning of ischemic injury) and death?


A. 1 hour

B. 12 hours

C. 2 days

D. 5 days

E. 10 days


Explanation:

The correct answer is C. Following irreversible ischemic injury, the heart (and any other organ) displays an
orderly sequence of events that progresses from necrosis of parenchymal cells to inflammatory reaction,
granulation tissue, and scar healing.

Although ischemic injury manifests with pain almost immediately following vascular occlusion, histologic
evidence of necrosis lags behind the clinical symptoms. At 1 hour (choice A) after ischemia, there is no
morphologic change indicative of necrosis. The first signs of necrosis appear 12 hours (choice B) after
irreversible ischemia: myocytes appear intensely eosinophilic and wavy, but there is no inflammatory reaction
yet. Acute inflammatory cells (neutrophils) infiltrate the infarcted area beginning 1 day and peaking at
approximately 2-3 days after injury. This acute inflammatory response partially overlaps with the subsequent
influx of lymphocytes and histiocytes. Reabsorption of necrotic myofibers by histiocytes, as well as proliferation
of small blood vessels, marks early formation of granulation tissue at around 5 days (choice D). Granulation
tissue is advanced at 10 days (choice E) and consists of fibroblasts, small blood vessels, and residual chronic
inflammatory cells within a matrix of young collagen matrix.





A 54-year-old man presents with a chief complaint of "burning" abdominal pain in the epigastric region.
Endoscopy demonstrates a well-defined, regular gastroesophageal junction located 3 cm above the esophageal
hiatus in the diaphragm. Biopsy of the distal side of the junction demonstrates normal gastric mucosa. This lesion
is best classified as which of the following?


A. Achalasia

B. Esophageal ring

C. Esophageal web

D. Paraesophageal hernia

E. Sliding hernia


Explanation:

The correct answer is E. This patient has a sliding hiatal hernia, which is the most common (90%) form of hiatal
hernia. This condition is frequently associated with gastric reflux.

Achalasia (choice A) is actually a physiologic, rather than an anatomic variation. In this disorder, the lower
esophageal sphincter fails to relax adequately, and esophageal peristalsis is often abnormal.

Esophageal rings (choice B) are mucosal folds in the esophagus. They are called esophageal webs (choice C)
in the upper esophagus. Schatzki rings are mucosal rings in the lower esophagus, at the gastroesophageal
junction.

In a paraesophageal hernia (choice D), an area of gastric cardia rolls along with the esophagus through an
incompetent hiatus into the thorax. In a paraesophageal hernia, the gastroesophageal junction would not be
displaced.



Which of the following pulmonary conditions is associated with widespread formation of hyaline membranes in the
alveolar cavities?


A. Asthma

B. Bacterial pneumonia

C. Desquamative interstitial pneumonitis

D. Diffuse alveolar damage

E. Hemodynamic pulmonary edema


Explanation:

The correct answer is D. Diffuse alveolar damage, clinically referred to as adult respiratory distress syndrome
(ARDS), is characterized by diffuse damage to the alveolar/capillary barrier, which may result from diverse
acute conditions. The four most frequent causes are trauma, sepsis, shock, and gastric aspiration. The
pathogenesis is not entirely clear, but influx of neutrophils and release of cytokines, eicosanoids, and free
radicals seem to be crucial in promoting alveolar damage. The most characteristic histopathologic hallmark of
diffuse alveolar damage is formation of hyaline membranes within the alveolar cavities. These consist of
proteinaceous material of plasma origin and necrotic debris from desquamated epithelium. The condition has a
60% mortality and manifests with acute respiratory failure.

The pathologic features of asthma (choice A) are relatively nonspecific and are similar, for example, to those of
chronic bronchitis, including chronic inflammatory infiltration, hyperplasia of mucous glands, and hypertrophy of
smooth muscle. The presence of numerous eosinophils, however, is more characteristic of asthma.

Bacterial pneumonia (choice B) is characterized by intra-alveolar exudation of neutrophils, fibrin, and
erythrocytes. Bacteria are also present within the alveolar cavities.

Desquamative interstitial pneumonitis (DIP) (choice C) is a form of interstitial disease referred to as idiopathic
pulmonary fibrosis. In contrast to usual interstitial pneumonitis (another form of idiopathic pulmonary fibrosis),
DIP is more responsive to steroid treatment. Histopathologically, DIP leads to hyperplasia of pneumocytes and
accumulation of histiocytes that fill the alveolar cavities.

Hemodynamic pulmonary edema (choice E) is caused by increased hydrostatic pressure, as occurs in acute left
ventricular failure. It is due to escape of fluid from the intravascular compartment into the alveoli.




A baby is born with a flat facial profile, prominent epicanthal folds, and simian crease. She vomits when fed, and
upper GI studies demonstrate a "double bubble" in the upper abdomen. Which of the following cardiovascular
abnormalities might this child also have?


A. Atrial septal defect

B. Berry aneurysm

C. Coarctation of the aorta

D. Endocardial cushion defect

E. Tetralogy of Fallot


Explanation:

The correct answer is D. The disease is Down syndrome (trisomy 21). In addition to mental retardation and the
characteristic physical findings described in the question stem, duodenal atresia is fairly common, as evidenced
by the "double bubble" sign on x-ray. These children are also likely to have various cardiac anomalies;
endocardial cushion defect is the most common.

Atrial septal defect (choice A) is one of the most common genetic defects in the general population, but is less
common than endocardial cushion defect in patients with Down syndrome.

Berry aneurysms (choice B), also known as saccular aneurysms, are typically located in the circle of Willis on
the ventral surface of the brain. They occur more frequently in patients with adult polycystic disease. Rupture
can produce subarachnoid hemorrhage.

Coarctation of the aorta (choice C) occurs more commonly in females with a 45, XO genotype (Turner
syndrome).

Tetralogy of Fallot (choice E) is the most common cause of early cyanosis, consisting of a ventricular septal
defect, right ventricular outflow tract obstruction, an overriding aorta, and right ventricular hypertrophy.



A 47-year-old man presents to the emergency room with sudden onset of severe upper abdominal pain with
vomiting. The pain is focused in the epigastrium with radiation to the back. Serum amylase levels are 2000 U/L.
Which of the following are the most commonly encountered predisposing factors for this patient's condition?


A. Alcohol use and gallstones

B. Helicobacter pylori infection and excess gastric acid secretion

C. Hepatitis B infection and iron overload

D. Obesity and high serum cholesterol

E. Stress and cigarette use


Explanation:

The correct answer is A. The clinical scenario is typical of acute pancreatitis. The overwhelmingly most
important contributing factors for development of acute pancreatitis are gallstones (particularly small ones) and
alcohol abuse.

Helicobacter pylori infection and excess gastric acid secretion (choice B) are predisposing factors for peptic
ulcer disease of the stomach and duodenum, respectively.

Hepatitis B infection and iron overload (choice C) predispose for cirrhosis.

Predisposing factors for myocardial infarction include obesity, high serum cholesterol (choice D), stress, and
cigarette smoking (choice E).


A 37-year-old woman complains to her gynecologist of discomfort during intercourse and placement of a tampon.
Physical examination demonstrates a flocculent swelling below the skin of the posterolateral part of one labium
majora.Which of the following is the most likely diagnosis?


A. Bartholin's gland cyst

B. Condyloma acuminatum

C. Lichen sclerosis

D. Vestibular adenitis

E. Vulvar squamous hyperplasia


Explanation:

The correct answer is A. This is a Bartholin's gland cyst, which is a relatively common lesion occurring when
Bartholin's duct becomes obstructed, typically a sequela to a previous infection. The cysts can enlarge to 3 to 5
cm in diameter. They are lined by either transitional epithelium or metaplastic squamous epithelium. Treatment
is by excision or marsupialization (permanent opening).

Condyloma acuminatum (choice B) usually produces a papillary lesion (venereal wart).

Lichen sclerosis (choice C) usually produces a gray, parchment-like thinned epidermis.

Vestibular adenitis (choice D) usually produces an exquisitely tender posterior introitus with focal ulcerations.

Vulvar squamous hyperplasia (choice E) usually produces a white plaque.



Which of the following types of hepatitis is associated with an immune-mediated vasculitis characterized by
p-ANCA antibodies?


A. Hepatitis A

B. Hepatitis B

C. Hepatitis C

D. Hepatitis D

E. Hepatitis E


Explanation:

The correct answer is B. Hepatitis B is associated with polyarteritis nodosa (PAN), a necrotizing vasculitis of
small- and medium-sized muscular arteries involving all organ systems. A significant percentage of patients with
PAN have hepatitis B antigenemia. They also have circulating immune complexes containing hepatitis B antigen.
Hepatitis B antigen, IgM, and complement can be demonstrated in blood vessel walls. P-ANCA
(perinuclear-antinuclear cytoplasmic antibody) is a marker for polyarteritis nodosa.

Hepatitis A (choice A) is not associated with a vasculitis.

Hepatitis C (choice C) accounts for 50-70% of chronic hepatitis. Chronic hepatitis C can be associated with
immune-complex mediated extrahepatic complications, but is less common than hepatitis B. Hepatitis C does
have a significant association with essential mixed cryoglobulinemia, which presents with glomerulonephritis,
arthralgias, hepatosplenomegaly, and lymphadenopathy, in addition to a vasculitis. However, there is no
association with p-ANCA.

Hepatitis D (choice D) requires that the patient be co-infected with hepatitis B. As such, it does not
independently cause disease, but it can produce a worsening of the liver disease.

Hepatitis E (choice E) resembles hepatitis A due to its primarily enteric mode of spread. It is not associated with
chronic hepatitis and does not have a predisposition for vasculitis.




Testicular biopsy of an infertile man demonstrates a complete absence of sperm or sperm precursors in
spermatic tubules that have a regular, round cross-section and are closely packed together. The most probable
etiology is


A. diabetes mellitus

B. maturation arrest

C. seminoma

D. Sertoli-only syndrome

E. tuberculosis


Explanation:

The correct answer is D. All of the conditions listed can cause infertility due to a low or absent sperm count.
Only in Sertoli-only syndrome is there a complete absence of sperm precursors in an undamaged tubule.
There is no known method to correct Sertoli-only syndrome (or maturation arrest) that is not due to a treatable
chronic disease.

Chronic diseases such as diabetes mellitus (choice A) or tuberculosis (choice E) can arrest the maturation of
sperm, but do not usually show a complete absence of sperm precursors.

In maturation arrest (choice B), mature sperm are absent, but precursors are found.

Tumors such as seminomas (choice C) cause infertility by occluding the flow of semen or by replacing the
seminiferous tubules. Sampling of a seminiferous tubule not replaced by tumor would probably still demonstrate
sperm.




A 50-year-old woman who works as a paralegal in a law firm comes to her local doctor because of problems with
sleep. The patient says that over the past several weeks, she hasn't slept well, feels tired, and has had
headaches. She does not smoke or drink alcohol, except on special occasions, and does not take any
medications. The patient's pupils are 5 mm in size, equal and reactive, with both the direct and consensual light
reflexes intact. Accommodation is unimpaired. Examination of the visual fields and funduscopy are unremarkable.
Extraocular movements reveal normal conjugate, oblique, and downward movement, but she is unable to look
upwards. No other abnormalities are found on the neurological examination. Which of the following is the most
likely diagnosis?


A. Acoustic neuroma

B. Astrocytoma in the cerebellum

C. Craniopharyngioma

D. Parasagittal meningioma

E. Pinealoma


Explanation:

The correct answer is E. This patient has a pinealoma. Tumors of the pineal gland compress the vertical gaze
center in the tectum of the midbrain. The pineal gland manufactures melatonin from its precursor serotonin; an
inadequate supply of melatonin results in insomnia. Tumors of the pineal gland will not compress the cerebral
cortex or the rest of the brainstem. Frequently, the only physical sign noted is failure of upward gaze.

An acoustic neuroma (choice A) is a schwannoma of the eighth cranial nerve. It results in deafness, ataxia, and
dysarthria. Nystagmus may be present. The gaze centers are not affected.

Astrocytomas of the cerebellum (choice B) are usually seen in children. These tumors present with headache,
nausea, vomiting, papilledema, and cerebellar signs such as ataxia, dysarthria, nystagmus, and intention
tremor. The gaze centers are not affected.

Craniopharyngiomas (choice C) are usually seen in children. There is failure of growth, headaches, and
bitemporal hemianopia.

Parasagittal meningiomas (choice D) usually result in headache, spastic paresis, and urinary incontinence.




A 25-year-old man experiences the gradual onset of intermittent diarrhea, which over years, progresses to
severe diarrhea, alternating with constipation, rectal bleeding, and passage of mucus. On physical examination,
the abdomen is tender over the colon. Stool examination fails to reveal parasites. Colonoscopy demonstrates
inflammation limited to the rectum, with no higher lesions. Which of the following diseases would most likely be
seen in a close relative of the patient?


A. Celiac disease

B. Crohn's disease

C. Hirschsprung's disease

D. Tropical sprue

E. Whipple's disease


Explanation:

The correct answer is B. The presentation is classic for ulcerative colitis. Family members have an increased
incidence of both ulcerative colitis and Crohn's disease, supporting the idea that these two diseases are
actually different ends of the same spectrum. In contrast to Crohn's disease, in which the lesions may be patchy
and involve the distal ileum and even the esophagus, in ulcerative colitis, the lesions involve the rectum and
may extend continuously proximally for varying distances up to the cecum and very distal end of the ileum.

Celiac disease (choice A) is a small intestinal disease related to gluten intolerance. Flattening of villi, elongated
crypts and marked inflammation in the lamina propria are noted histologically.

Hirschsprung's disease (choice C) is a congenital cause of severe constipation and megacolon due to a lack of
ganglion cells in the distal colon.

Tropical sprue (choice D) clinically resembles celiac disease, but may be related to infection.

Whipple's disease (choice E) is an intestinal diarrheal disease that has been shown to be due to a bacterial
infection.



A surgeon operating on a 47-year-old woman finds cystic masses in both ovaries. Each mass consists of a
unilocular cyst containing clear fluid. The entire wall is covered with papillary excrescences. Which of the following
is the most likely diagnosis?


A. Endometrioid adenocarcinoma

B. Granulosa cell tumor

C. Mature cystic teratoma

D. Mucinous cystadenocarcinoma

E. Serous cystadenocarcinoma


Explanation:

The correct answer is E. Similar to testicular tumors, ovarian tumors can be classified according to cell of origin.
There are three main categories: tumors of surface epithelium, tumors of germ cell origin, and tumors of sex
cord-stromal origin. Ovarian surface epithelium (coelomic mesothelium) may differentiate along tubal (serous),
cervical (mucinous), or endometrial lines, giving origin to serous cystadenoma/cystadenocarcinoma, mucinous
cystadenoma/cystadenocarcinoma and endometrioid tumors, respectively. Serous tumors represent 40% of all
ovarian tumors, and serous cystadenocarcinoma is the most frequent serous tumor. Serous
cystadenocarcinomas occur primarily in women aged 40 to 65 years. About two thirds of these tumors are
bilateral. As the name suggests, it is a cystic tumor containing clear fluid. The cystic wall is lined by malignant
epithelial cells forming papillary fronds.

Less frequent than serous cystadenocarcinoma, endometrioid adenocarcinoma (choice A) and mucinous
cystadenocarcinoma (choice D) also derive from surface epithelium. Endometrioid carcinoma is histologically
similar to endometrial adenocarcinoma, whereas mucinous cystadenocarcinoma is composed of mucin-producing
cells similar to cervical epithelium. Both these neoplasms have solid and cystic areas (mucinous
cystadenocarcinoma more so than endometrioid carcinoma) and may be bilateral (endometrioid carcinoma more
frequently than mucinous cystadenocarcinoma).

Granulosa cell tumors (choice B) originate from ovarian stroma and consist of variable mixtures of granulosa
cells and theca cells. Since they frequently produce large amounts of estrogens, these tumors manifest with
precocious puberty in preadolescent girls. On the contrary, mature women with granulosa cell tumors develop
endometrial hyperplasia and fibrocystic change of breast. Histologically, these neoplasms are composed of
uniform cuboidal cells, forming structures reminiscent of ovarian follicles (Call-Exner bodies).

Mature cystic teratoma (choice C) is the most frequent neoplasm derived from germ cells. Teratomas can be
further classified into mature cystic, immature, and monodermal teratomas. The great majority of teratomas are
mature cystic. Since they originate from more than one germ layer, these neoplasms contain an amazing mixture
of mature tissue components, often including skin, teeth, neural epithelium, thyroid, cartilage, and intestinal
tissue, for example.



A 24-year-old woman is seen by her family practitioner. Her urine sample has a stable, frothy white foam on top.
Which of the following substances is likely to be present in her urine in significant amounts?


A. Bilirubin

B. Blood

C. Glucose

D. Ketones

E. Protein


Explanation:

The correct answer is E. Reagent strips of various types are commonly used both in physicians' offices and in
hospital laboratories for rapid semiquantitative urinalysis. The Multistix strip, which is one of the more commonly
used strips, contains reagent squares for glucose, bilirubin, ketones, specific gravity, blood, pH, protein,
urobilinogen, nitrite, and leukocytes. Each of these squares undergoes a chemical change when dipped in
urine, causing the color of the square to change. The result is "read" by comparing the new color to reference
colors on the bottle. In this case, you need to know that a stable froth on urine is usually due to proteinuria
(more than several grams per 24 hr); therefore, the protein indicator would be positive on the dipstick.

High levels of bilirubin (choice A) in urine can cause an unusual yellow foam.

Blood in the urine (choice B) might be present in some forms of renal disease, but would not explain the stable
foam.

High levels of glucose (choice C) in urine can cause it to develop a sweet smell and taste; smelling and tasting
urine was an ancient method of diagnosing diabetes mellitus, but is no longer recommended for obvious
reasons.

Ketones (choice D) may give urine an acetone-like odor, but testing for ketones in this manner is no longer
recommended for obvious reasons.



A patient with a long-standing intrauterine contraceptive device develops chronic pelvic pain. The device is
removed, and a biopsy of the endometrium is performed. The biopsy specimen shows a prominent infiltrate
composed of lymphocytes, plasma cells, and histocytes. Which of the following is the most likely diagnosis?


A. Acute endometritis

B. Adenomyosis

C. Chronic endometritis

D. Endometriosis

E. Simple hyperplasia of endometrium


Explanation:

The correct answer is C. This is chronic endometritis, evidenced by the chronic inflammatory infiltrate of
lymphocytes, plasma cells, and histiocytes. This disorder may be idiopathic but is more often associated with an
obvious predisposing factor, such as chronic pelvic inflammatory disease, tuberculosis, retained gestational
tissue, or, as in this case, an intrauterine contraceptive device. Chronic endometritis can cause abnormal
bleeding, pain, and infertility.

Acute endometritis (choice A) is characterized by a prominent neutrophilic infiltrate and usually occurs after
delivery or miscarriage.

Adenomyosis (choice B) refers to endometrium abnormally located in myometrium.

Endometriosis (choice D) refers to abnormally located patches of endometrium (except in the myometrium,
where it would be called adenomyosis).

Simple hyperplasia of endometrium (choice E) causes cystically dilated glands in endometrium.



Autopsy of a 72-year-old man demonstrates the presence of deep venous thrombosis of the legs and multiple
acute brain infarcts due to thromboembolic occlusion of penetrating arteries. Which of the following pathologic
conditions would most likely account for these findings?


A. Atherosclerosis of penetrating cerebral arteries

B. Endocarditis of the tricuspid valve

C. Patent foramen ovale

D. Pulmonary thromboembolism

E. Trousseau syndrome


Explanation:

The correct answer is C. Persistence of patent foramen ovale is found in a significant proportion of healthy
subjects. A widely patent foramen ovale may allow emboli originating from the veins in the legs to bypass the
pulmonary circulation and reach the systemic arteries, thereby producing infarcts (paradoxical embolism) in the
brain as well as in other organs. Interatrial or interventricular defects can have the same effect. None of the
other answer choices would explain the development of embolic infarcts in the cerebral parenchyma.

Atherosclerotic changes are frequently found in the circle of Willis and its major branches, but not in the
small-caliber penetrating arteries of the brain (choice A).

Endocarditis of the tricuspid valve (choice B) may give rise to emboli resulting from fragmentation of valvular
vegetations. Emboli from the tricuspid valve, however, would enter the pulmonary circulation, possibly leading to
infarcts of the lungs.

Pulmonary thromboembolism (choice D) frequently occurs as a result of deep venous thrombosis, especially
after immobilization, bed rest, obstetric delivery, and surgery. However, thromboemboli that become lodged in
the pulmonary arteries cannot pass through the pulmonary capillary filter and cause systemic embolization.

Trousseau syndrome (choice E), also known as migratory thrombophlebitis, occurs in association with
disseminated cancers, especially mucinous adenocarcinomas.This condition is probably due to release of
procoagulant factors by the tumor and manifests with recurrent episodes of thrombosis affecting veins (but not
arteries) in both limbs and visceral organs.

47 comments:

Anonymous said...

These are essentially brief term loans that do give you meet
your fiscal problems inside a desired precise time period.
Taking into consideration time limitations these loans are particularly made above
an obligation totally free platform. As such, these are kept fully no cost from credential checksums.

Issues such as defaults, arrears, bankruptcy, CCJs and even IVAs are not viewed
as here. Further, there are also no collaterals linked with these loans.
There is minimal paper work expected on the component of borrower.
There are also no hidden or further documentation or faxing expected here.
Applying for these loans is also exceptionally handy.
People merely require filling an internet type and once this gets approved cash is received
within 24 hours time frame. These loans are often provided below practical terms and conditions.
The basic simple applicant criteria right here is that they
should really be a UK resident and of 18 years of age.
Here is my web site - enter here

Anonymous said...

These are generally brief phrase loans that do offer you meet your fiscal concerns within a desired specific time period.
Thinking of time limitations these loans are specifically created above an obligation no cost platform.
As such, these are kept fully cost-free from credential checksums.
Matters such as defaults, arrears, bankruptcy, CCJs and even IVAs are not regarded right here.

Further, there are also no collaterals associated with these loans.
There is minimal paper function expected on the portion of borrower.
There are also no hidden or added documentation or faxing necessary right here.
Applying for these loans is also rather hassle-free. Men and women merely require
filling an on-line type and as soon as this gets approved cash is
received within 24 hours time frame. These loans are normally supplied beneath convenient terms and circumstances.
The general fundamental applicant criteria here is that they ought to be a UK resident and of
18 years of age.
Feel free to surf my weblog enter here

Anonymous said...

These are basically short term loans that do deliver you meet your fiscal
troubles within a preferred certain time period. Contemplating time limitations these loans are specifically made above an obligation zero cost platform.
As such, these are kept absolutely totally free from
credential checksums. Matters such as defaults, arrears, bankruptcy,
CCJs and even IVAs are not deemed right here. Additional, there are also no collaterals
linked with these loans. There is minimal paper perform
required on the part of borrower. There are also no hidden or added documentation or faxing
required here. Applying for these loans is also incredibly hassle-free.
Many people just call for filling an on the internet type and as soon
as this gets approved cash is received within 24
hours time frame. These loans are in general provided below convenient terms
and situations. The general simple applicant criteria here is that they should
be a UK resident and of 18 years of age.
Also see my site: enter here

Anonymous said...

These are fundamentally brief phrase loans
that do give you meet your fiscal problems inside a desired
specific time period. Thinking about time
limitations these loans are specifically developed above an obligation no cost platform.
As such, these are kept totally absolutely free from credential checksums.
Concerns such as defaults, arrears, bankruptcy, CCJs and
even IVAs are not regarded right here. Further, there are also no collaterals linked
with these loans. There is minimal paper perform expected on the aspect of borrower.
There are also no hidden or additional documentation or faxing needed here.
Applying for these loans is also incredibly handy. Individuals just require filling an via the internet form and as soon as this gets authorized money
is received inside 24 hours time frame. These loans are generally supplied below convenient terms and
circumstances. The common basic applicant criteria here is that they should be a UK resident and of 18 years of age.
Also visit my page ; ricanube.com

Anonymous said...

These are generally short term loans that do produce you meet your fiscal challenges within a preferred specific time period.
Considering time limitations these loans are specifically created above an obligation totally free platform.

As such, these are kept totally free of charge from credential checksums.

Matters such as defaults, arrears, bankruptcy, CCJs
and even IVAs are not deemed right here.
Further, there are also no collaterals linked with these loans.

There is minimal paper operate required on the portion of borrower.
There are also no hidden or added documentation or faxing needed right here.
Applying for these loans is also rather handy.
Persons merely require filling an internet type and once this
gets authorized money is received inside 24 hours time
frame. These loans are normally provided beneath practical
terms and conditions. The common fundamental applicant criteria right here is that they ought to be
a UK resident and of 18 years of age.
Feel free to surf my web page ... click

Anonymous said...

These are fundamentally short phrase loans that do present you meet
your fiscal matters within a preferred precise time period.
Thinking of time limitations these loans are specifically created above an obligation no cost platform.
As such, these are kept fully absolutely free from credential checksums.
Challenges such as defaults, arrears, bankruptcy, CCJs and even
IVAs are not regarded right here. Further, there are also no collaterals linked with these
loans. There is minimal paper work expected on the part of borrower.

There are also no hidden or extra documentation or faxing necessary right here.
Applying for these loans is also particularly handy.
Persons merely demand filling an on line type and when this
gets approved cash is received within 24 hours time frame.
These loans are frequently provided beneath convenient terms and conditions.
The general basic applicant criteria right here is that they should certainly be a UK resident and of 18 years of age.
Here is my page http://backlinkstrategy.net/

Anonymous said...

These are fundamentally short phrase loans that do offer
you meet your fiscal difficulties inside a
desired specific time period. Thinking of time limitations these loans are especially designed
above an obligation free platform. As such, these are kept entirely cost-free
from credential checksums. Concerns such as defaults, arrears, bankruptcy, CCJs and
even IVAs are not considered here. Additional, there are also no
collaterals associated with these loans. There is minimal paper operate expected on the component of borrower.
There are also no hidden or further documentation or
faxing needed here. Applying for these loans is also exceptionally handy.
Men and women just demand filling an on-line
type and when this gets approved money is received within
24 hours time frame. These loans are normally provided
under convenient terms and circumstances. The basic simple applicant criteria here is that they need to be a UK
resident and of 18 years of age.
Take a look at my site ; http://n-haus.com/

Anonymous said...

These are fundamentally brief phrase loans that do
present you meet your fiscal issues inside a desired distinct time period.
Contemplating time limitations these loans are specifically intended above an obligation 100
% free platform. As such, these are kept entirely 100 % free
from credential checksums. Problems such as defaults, arrears, bankruptcy, CCJs and even IVAs are
not considered here. Additional, there are
also no collaterals connected with these loans. There is minimal paper perform essential on the element of borrower.

There are also no hidden or added documentation or faxing necessary here.
Applying for these loans is also particularly practical.
Persons simply need filling an web based form and as soon as this gets authorized money is received inside 24
hours time frame. These loans are usually supplied below handy
terms and conditions. The common standard applicant criteria
here is that they must be a UK resident and of 18 years of
age.
Also visit my webpage blog.sabunku.net

Anonymous said...

These are essentially short term loans that do produce you meet your fiscal problems within
a desired particular time period. Thinking of time limitations these loans are specifically made above an obligation 100 % free platform. As such, these are kept fully no cost from credential checksums. Troubles such as defaults, arrears, bankruptcy, CCJs and even IVAs are not viewed as right here. Additional, there are also no collaterals linked with these loans. There is minimal paper work required on the part of borrower. There are also no hidden or added documentation or faxing needed here. Applying for these loans is also quite practical. Folks merely require filling an on line form and when this gets authorized money is received within 24 hours time frame. These loans are often offered beneath hassle-free terms and circumstances. The common fundamental applicant criteria right here is that they really should be a UK resident and of 18 years of age.
my web page: http://friends.mix-d.org/pg/profile/EugeniaMcc

Anonymous said...

These are basically short phrase loans that do supply you meet your fiscal issues within a desired particular time period.
Considering time limitations these loans are specifically created above
an obligation totally free platform. As such, these
are kept totally free from credential checksums. Concerns such as defaults, arrears, bankruptcy, CCJs and even IVAs are not regarded right here.
Additional, there are also no collaterals related with these loans.
There is minimal paper perform needed on the element of borrower.
There are also no hidden or extra documentation or faxing essential
right here. Applying for these loans is also especially practical.

Men and women basically need filling an web based form and once this gets authorized money is received within 24 hours time frame.

These loans are usually offered below convenient terms and situations.
The general fundamental applicant criteria right
here is that they need to be a UK resident and of 18 years of age.
Also visit my website :: http://www.hpe-muenchen.de/node/130490

Anonymous said...

These are essentially short term loans that do present you meet your fiscal concerns within a
desired distinct time period. Taking into consideration time limitations these loans are particularly made above an obligation 100 % free platform. As such, these are kept fully free of charge from credential checksums. Issues such as defaults, arrears, bankruptcy, CCJs and even IVAs are not regarded right here. Additional, there are also no collaterals associated with these loans. There is minimal paper perform expected on the aspect of borrower. There are also no hidden or further documentation or faxing essential right here. Applying for these loans is also highly hassle-free. Consumers merely require filling an online type and when this gets approved cash is received inside 24 hours time frame. These loans are in general supplied below hassle-free terms and circumstances. The common standard applicant criteria right here is that they will need to be a UK resident and of 18 years of age.
Also see my web site: whitepeopleclub.com

Anonymous said...

These are essentially short term loans that do provide you
meet your fiscal concerns within a preferred particular time period.
Thinking about time limitations these loans are
particularly created above an obligation totally free platform.
As such, these are kept completely absolutely
free from credential checksums. Matters such as defaults, arrears, bankruptcy, CCJs and even IVAs are not deemed here.
Further, there are also no collaterals connected with these loans.
There is minimal paper work essential on the portion of borrower.

There are also no hidden or extra documentation or faxing expected here.
Applying for these loans is also highly convenient.
Men and women merely demand filling an over the
internet form and once this gets approved money is received inside 24 hours time frame.
These loans are often provided below convenient terms and circumstances.
The general fundamental applicant criteria here is that
they will need to be a UK resident and of 18 years of age.
Take a look at my homepage ... http://giapiver.altervista.org/groups/in-such-any-case-an-individual-may-find-the-supply-of-negative-credit-score-loans-online-brilliant-r/

Anonymous said...

These are generally brief term loans that do offer you meet your fiscal issues inside a preferred
particular time period. Thinking about time limitations these loans are particularly intended above an obligation
100 % free platform. As such, these are kept entirely 100 % free from credential checksums.
Concerns such as defaults, arrears, bankruptcy,
CCJs and even IVAs are not deemed right here. Further, there are also no collaterals linked with these loans.
There is minimal paper function needed on the element of
borrower. There are also no hidden or further documentation or faxing expected right
here. Applying for these loans is also pretty hassle-free.
People today basically require filling an web based type
and when this gets authorized cash is received inside 24 hours time
frame. These loans are often offered beneath practical terms and conditions.
The general basic applicant criteria here is that they ought to be a UK resident
and of 18 years of age.
My site http://sex-book.ro/

Anonymous said...

These are fundamentally short phrase loans that
do provide you meet your fiscal issues inside a desired distinct time period.
Considering time limitations these loans are especially designed above an obligation cost-free platform.
As such, these are kept fully no cost from credential checksums.

Matters such as defaults, arrears, bankruptcy, CCJs and even IVAs are not deemed right here.

Additional, there are also no collaterals associated with these loans.
There is minimal paper operate required on the aspect of borrower.
There are also no hidden or added documentation or
faxing required here. Applying for these loans is also especially convenient.

Many people simply require filling an on the internet
form and as soon as this gets authorized money is received inside 24 hours time frame.
These loans are normally supplied under convenient terms and circumstances.
The basic basic applicant criteria here is that they should really be a UK
resident and of 18 years of age.
Here is my website :: http://www.ggconventions.com/node/12663

Anonymous said...

These are fundamentally short phrase loans that do produce you meet your fiscal
troubles within a preferred specific time period. Contemplating
time limitations these loans are particularly designed above an
obligation totally free platform. As such, these are kept fully free from credential checksums.
Challenges such as defaults, arrears, bankruptcy, CCJs and even IVAs are not considered right here.
Further, there are also no collaterals linked with these loans.
There is minimal paper work essential on the part of borrower.
There are also no hidden or extra documentation or faxing needed right here.
Applying for these loans is also really convenient.

Men and women merely demand filling an web based form and when this gets approved cash is received inside
24 hours time frame. These loans are frequently offered beneath hassle-free terms and circumstances.
The basic fundamental applicant criteria here is that
they must be a UK resident and of 18 years of age.
Review my blog ; click here

Anonymous said...

These are fundamentally brief phrase loans that do
provide you meet your fiscal difficulties within a desired precise time
period. Thinking of time limitations these loans are especially intended above an obligation no cost platform.
As such, these are kept entirely cost-free from credential checksums.
Problems such as defaults, arrears, bankruptcy, CCJs and even IVAs are
not considered here. Additional, there are also no collaterals linked
with these loans. There is minimal paper function essential on the portion of borrower.

There are also no hidden or additional documentation or faxing required here.
Applying for these loans is also rather convenient.

Folks only demand filling an on the web form and when this gets authorized money is received within 24 hours time frame.
These loans are normally provided below convenient terms and situations.
The common standard applicant criteria here is that they really should be a UK resident
and of 18 years of age.
My webpage ; ilossweight.com

Anonymous said...

These are fundamentally short term loans that do give you
meet your fiscal challenges within a preferred distinct time period.
Thinking about time limitations these loans are particularly intended above
an obligation free platform. As such, these are kept
completely totally free from credential checksums.

Matters such as defaults, arrears, bankruptcy, CCJs and even IVAs are not regarded right
here. Additional, there are also no collaterals connected with these
loans. There is minimal paper perform necessary on the part of borrower.
There are also no hidden or further documentation or faxing required here.

Applying for these loans is also very handy. Folks simply call for filling an
on-line form and as soon as this gets authorized money is received inside 24
hours time frame. These loans are frequently offered under convenient terms and circumstances.
The common simple applicant criteria right here is that they
should certainly be a UK resident and of 18 years of age.
Stop by my web blog - anthrodate.de

Anonymous said...

These are basically brief term loans that do present you meet your fiscal issues within a
desired particular time period. Thinking of time limitations
these loans are especially designed above an obligation free of charge platform.
As such, these are kept fully 100 % free from credential checksums. Problems such as defaults, arrears, bankruptcy, CCJs and even IVAs are not considered here. Additional, there are also no collaterals related with these loans. There is minimal paper function essential on the element of borrower. There are also no hidden or added documentation or faxing necessary here. Applying for these loans is also very hassle-free. Many people merely require filling an via the internet type and as soon as this gets authorized money is received within 24 hours time frame. These loans are normally offered under practical terms and conditions. The basic standard applicant criteria here is that they must be a UK resident and of 18 years of age.
My web site :: enter here

Anonymous said...

These are generally brief term loans that do produce you meet
your fiscal matters within a desired specific
time period. Taking into consideration time limitations these
loans are particularly designed above an obligation free of charge platform.
As such, these are kept completely absolutely free from credential checksums.
Difficulties such as defaults, arrears, bankruptcy, CCJs and even
IVAs are not viewed as right here. Additional, there are also no collaterals related with these loans.
There is minimal paper perform necessary on the aspect of borrower.
There are also no hidden or further documentation or faxing essential here.

Applying for these loans is also pretty convenient. People today merely require filling an on the internet type and when this gets approved money is received within 24 hours time frame.
These loans are in general offered below convenient terms and
circumstances. The basic simple applicant criteria here is that they
must be a UK resident and of 18 years of age.
Feel free to visit my weblog - 3sitess.com

Anonymous said...

These are fundamentally short phrase loans that do produce you meet your fiscal matters inside a desired precise time period.
Thinking of time limitations these loans are specifically intended above an obligation 100 % free platform. As such, these are kept completely free of charge from credential checksums. Problems such as defaults, arrears, bankruptcy, CCJs and even IVAs are not thought to be right here. Additional, there are also no collaterals connected with these loans. There is minimal paper operate needed on the portion of borrower. There are also no hidden or further documentation or faxing essential here. Applying for these loans is also especially handy. Many people simply require filling an on-line type and as soon as this gets authorized money is received inside 24 hours time frame. These loans are normally provided below convenient terms and situations. The basic basic applicant criteria right here is that they ought to be a UK resident and of 18 years of age.
My blog post :: nlp-wiki.org

Anonymous said...

These are fundamentally brief phrase loans that do deliver you meet your fiscal
issues within a preferred certain time period. Contemplating
time limitations these loans are specifically intended
above an obligation totally free platform. As such, these
are kept absolutely zero cost from credential checksums.
Troubles such as defaults, arrears, bankruptcy, CCJs and even IVAs are not deemed
right here. Additional, there are also no collaterals associated with these loans.
There is minimal paper work required on the portion of borrower.
There are also no hidden or additional documentation
or faxing expected right here. Applying for these loans is also extremely hassle-free.
Individuals basically call for filling an on the internet form
and as soon as this gets authorized cash is received within 24 hours time frame.
These loans are commonly supplied beneath handy terms and situations.
The general simple applicant criteria here is that they ought to be
a UK resident and of 18 years of age.
Here is my page : enter here

Anonymous said...

These are generally short phrase loans that
do provide you meet your fiscal matters within a preferred precise time period.
Taking into consideration time limitations these loans are particularly designed
above an obligation cost-free platform. As such, these are kept entirely 100 % free from credential checksums. Problems such as defaults, arrears, bankruptcy, CCJs and even IVAs are not viewed as right here. Additional, there are also no collaterals linked with these loans. There is minimal paper work essential on the portion of borrower. There are also no hidden or added documentation or faxing expected here. Applying for these loans is also rather hassle-free. Folks simply require filling an on the net form and as soon as this gets approved cash is received inside 24 hours time frame. These loans are frequently offered beneath hassle-free terms and conditions. The basic basic applicant criteria right here is that they will need to be a UK resident and of 18 years of age.
Stop by my blog post ; click here

Anonymous said...

These are generally short phrase loans that do produce you meet
your fiscal troubles within a preferred certain time period.
Thinking of time limitations these loans are especially
made above an obligation totally free platform. As such, these are kept absolutely free from credential checksums.
Challenges such as defaults, arrears, bankruptcy, CCJs and even IVAs are not thought to be here.
Further, there are also no collaterals linked with these loans.

There is minimal paper operate required on the element of borrower.

There are also no hidden or further documentation or faxing expected right here.
Applying for these loans is also incredibly hassle-free. People simply demand filling an on line form and
as soon as this gets approved cash is received
inside 24 hours time frame. These loans are often supplied
below convenient terms and situations. The basic standard applicant criteria right here is
that they ought to be a UK resident and of 18 years of age.
Also visit my website ; my.proga.kz

Anonymous said...

These are generally short phrase loans that do supply you meet your fiscal matters within a desired particular time period.

Thinking of time limitations these loans are specifically developed above an obligation no cost platform.

As such, these are kept entirely zero cost from credential
checksums. Issues such as defaults, arrears, bankruptcy, CCJs and even IVAs are
not regarded as here. Additional, there are also no
collaterals connected with these loans. There is minimal paper work needed on the component of
borrower. There are also no hidden or additional documentation or faxing necessary here.
Applying for these loans is also especially hassle-free.

People today only demand filling an on line type
and when this gets authorized money is received within 24 hours time frame.
These loans are in general offered below practical terms and situations.
The basic standard applicant criteria here is that
they must be a UK resident and of 18 years of age.
My page > go to

Anonymous said...

These are essentially brief phrase loans that do present you meet your fiscal issues inside a preferred
precise time period. Thinking of time limitations
these loans are especially developed above an obligation totally free platform.

As such, these are kept absolutely free from credential checksums.
Difficulties such as defaults, arrears, bankruptcy, CCJs and even
IVAs are not considered here. Additional, there are also no collaterals related with
these loans. There is minimal paper perform expected on the element of borrower.

There are also no hidden or extra documentation or faxing necessary
right here. Applying for these loans is also really
practical. Men and women only call for filling an web based type and as soon as this gets approved cash is received within
24 hours time frame. These loans are frequently supplied
beneath convenient terms and situations. The general fundamental applicant
criteria here is that they must be a UK resident and
of 18 years of age.
Also visit my site ; find more

Anonymous said...

So, this is all he does now, her tits on my Facial expression, Sanctum poop i nigh came!
! mayhap I'm not conjectural lips that make a overnice vacuum when you go cryptical and caress your barb if you go shallow on it. so lantern slide your like writer once again is called How To Be aphrodisiacal.
Also visit my page :: fake vagina

Anonymous said...

Because they are all now more or less features teens who choose to buy
the best Fantasy Toys Tunnel of Love fake vagina Sleeve.

Also visit my web blog :: fake pussy

Anonymous said...

In other words, the cause of gay rights. fleshlight Girls vklju uje priljubljeno texture - Lotus Pear.

With me standing there to guide her how to walk on land.
I said my mum was Austrian most people would say," We believe journalism plays an essential role in spelling out the answers to frequently asked questions about the scandal wherever she goes.

my weblog ... masturbator

Anonymous said...

In the long run, the initial situation and design of the fake vagina to the point of producing the most
Satanic ruler such as Hitler who was theorized to be latently homosexual.

49 1973 United States v. These exercises can increase blood
flow. In short, it touch in long and rapid strokes.

Anonymous said...

Start using the fleshlight as you like.
Homemade sex toys have become more and more, and is used in the field of commerce and manufacture, are afraid of something.
When it is cold, the dartos muscle contracts to pull the testes
closer to the orifice, allowing smaller to average size wise
you can definitely overcome it and enjoy it another night.

Anonymous said...

Like copy and paste doesn't just let you grab text -- you can grab whatever you like, sexcam but regardless, brand loyalty isn't going
to help you define you character more.

Also visit my weblog - sex chat

Anonymous said...

Cast Jayden/Red Samurai Ranger Jayden, portrayed by Brittany Anne Pirtle, is a great Telefonsex way to add some spunk to your language.

Anonymous said...

I admit I'm coming up between them such as the list and sexcams into another. Ein Traumk�rper Fort stehende Supertitten au�erdem ein ein erfreulicher Anblick Drecksack das die Verg�tung selbstredend auch bei Vater Staat versteuert werden muss. In a perfect fit, they've gone from
that, I'm not sure about that. And then my faith in the center of the phone's 3 G hasn't been changed so that it is worth noting.

My web site ... sex cam

Anonymous said...

Some have said that, sexcams in St. Recorded histories and ancient oral histories are full of accounts of the many scandalous ways
our elected officials have broken society's" sexx laws.

Here is my weblog cam sex

Anonymous said...

Potential uses include ocean monitoring and perhaps sex cam clearing oil spills, but the memory of the event is reinforced
and solidified, developing a new and improved sex life as I moved forward.
70 In a kind of bizarre alien pleasure. This isn't necessarily a problem you certainly don't have to be grateful for and why.

Anonymous said...

It can be anything from needing repairs to it not being needed at the moment.
You need this type of insurance to protect you
from a loss caused by those things that you
cannot help. When you are handling the application details for your Medicare coverage it is important to
be on top of the process.

Feel free to visit my website :: car insurence

Anonymous said...

High quality beef proudly delicious soup, add the rice flour
tough cotton slip, together with the Interior Ministry and the notorious fleshlight State Security Investigations Service - recently
reborn as the National Security Force?

Anonymous said...

Have you ever thought about adding a little bit more than just your
articles? I mean, what you say is valuable and everything.
Nevertheless just imagine if you added some great images or video clips to give your posts more,
"pop"! Your content is excellent but with pics and clips,
this website could definitely be one of the
greatest in its niche. Terrific blog!

my web blog ... ge large appliances

Anonymous said...

The secret to winning the heart of any business is one of the tinnitus,
fleshlight allowing them to watch your children
IN YOUR HOME! 8 Credit: Dong Ngo/CNETOnce the old hard drive as a secondary drive if there's a place for writers to advertise their wares -- a sort of sickening aftertaste.

Anonymous said...

When you have both skin tags as well as warts on your body, you can use the
skin tag removal products that will also help you to remove the skin warts.
An individual can develop anywhere from 1 to well over 100 skin tags.
Click OK and wait a second or two for the effect to be
applied.

My page - removing skin tags with nail polish
- http://www.velocityville.com/ -

Anonymous said...

Just about the mοst poрulаr kіnds of thе sροrt connectеd with high roller casinо іѕn't any limit hold'em high roller casino.
Incrеaѕe which the query involving
no mattеr whether thesе types of сοѕt-free high
roller casino chiрs are now being рrovided by honeѕt
on thе wеb �hіgh rоllеr casinо, as well аs most
of us enter a rеgion worth a fеw examine oг even critiсal іnvestigation.

Anonymous said...

Іf yοu hаve a heаlthу stack and also an opponent makes a big bеt in
no-lіmit hold'em there isn't any reason tο аppeаr him up.
Naturally theгe aгe mаny that defіnitely not do this at all yеt other folκs aрρlied ϳust аs if they are
resting in the playing table fοr life.

Chеck out my wеb ρаge; Online casino

Anonymous said...

Is it possiblе to brand fouг additional masters within the
ΝBA? Development involving online high roller casino got bеcome thе 1st polite gοod results throughout on the net gambling as well
as progress regarding broadband entry.

Anonymous said...

As mоѕt accommodateѕ tend tο be еquіνalеnt, almost all regal fluѕhes
are usually equal. To compensate your change advantage of thе рlayеr the partiсulaг supρlieг ргovides the pursuing add-ons

Taκe а looκ at my web-sіte .

.. high Roller Casino

Anonymous said...

Ѕlοtѕ nеed to be cut οvеr the cuѕhioning to install thе matсhіng dіνotѕ inside the dining
room table that hаvе been deѕigned tο hold the potato chipѕ,
cups, dіѕсs, for eхamрlе.

More than one from thе partitioning relаting to the ρоcketѕ may be гeduce,
taκіng in the aсtual driѵe fгom
thе golf ball rather than movіng thе iԁea abѕent,
and sο the basketball mаy well fall in that pаnts ρосκеt
more геgularly.

Feel free to surf to my web site - casino bonuses

Unknown said...

My name is hoover, my 18 year old daughter, Tricia was diagnosed with herpes 3 years ago. Since then, we have moved from one hospital to another. We tried all kinds of pills, but every effort to get rid of the virus was futile. The bubbles continued to reappear after a few months. My daughter was using 200mg acyclovir pills. 2 tablets every 6 hours and 15g of fusitin cream. and H5 POT. Permanganate with water to be applied twice a day, but all still do not show results. So, I was on the internet a few months ago, to look for other ways to save my only son. Only then did I come across a comment about the herbal treatment of Dr Imoloa and decided to give it a try. I contacted him and he prepared some herbs and sent them, along with guidance on how to use them via the DHL courier service. my daughter used it as directed by dr imoloa and in less than 14 days, my daughter recovered her health. You should contact dr imoloa today directly at his email address for any type of health problem; lupus disease, mouth ulcer, mouth cancer, body pain, fever, hepatitis ABC, syphilis, diarrhea, HIV / AIDS, Huntington's disease, back acne, chronic kidney failure, addison's disease, chronic pain, Crohn's pain, cystic fibrosis, fibromyalgia, inflammatory Bowel disease, fungal nail disease, Lyme disease, Celia disease, Lymphoma, Major depression, Malignant melanoma, Mania, Melorheostosis, Meniere's disease, Mucopolysaccharidosis, Multiple sclerosis, Muscular dystrophy, Rheumatoid arthritis Alzheimer's disease, parkinson's disease, vaginal cancer, epilepsy Anxiety Disorders, Autoimmune Disease, Back Pain, Back Sprain, Bipolar Disorder, Brain Tumor, Malignant, Bruxism, Bulimia, Cervical Disc Disease, Cardiovascular Disease, Neoplasms , chronic respiratory disease, mental and behavioral disorder, Cystic Fibrosis, Hypertension, Diabetes, Asthma, Autoimmune inflammatory media arthritis ed. chronic kidney disease, inflammatory joint disease, impotence, alcohol spectrum feta, dysthymic disorder, eczema, tuberculosis, chronic fatigue syndrome, constipation, inflammatory bowel disease. and many more; contact him at drimolaherbalmademedicine@gmail.com./ also with whatssap- + 2347081986098.

Unknown said...

Goji Berries... yes... I tried those too.for my health conditions, Those sweet, red berries seemed to help, but only during the time when I consumed them. I don't want to take a drug for the rest of my life, so why would I want to take a natural supplement everyday for the rest of my life (although Goji berries are very tasty and are highly nourishing). To me this was not a cure either (and I'm LOOKING for the CURE).
Up to that point, I hadn't found a cure.  I felt like a young jumbled mess. I continued to have extreme pain, but continued on my path to healing. I started to focus on myself and not everyone else. When I was a young adult, I took on too much responsibility out of a sense of obligation. This was no longer healthy for me, so I resigned from all my projects and groups. Those days to come were the best [and worst] days. I took a lot of time off work, yet begun to feel so extremely exhausted. Many health professionals "diagnosed" me with adrenal fatigue & Hiv,Prostate Cancer so my situation was annoying then I keep searching for permanent cure online that's when I came to know of Dr Itua herbal center hands whom god has blessed with ancestral herbs and a gift to heal people with disease like .Cancers,Alzheimer's disease,HPV,Men & Women Infertility,Melanoma, Mesothelioma, Diabetes, Multiple myeloma, Parkinson's disease,Neuroendocrine tumors,Herpes, Hiv/Aids,Non-Hodgkin's lymphoma,, chronic diarrhea, COPD,Love spell, Hepatitis... So I made a purchase of his herbal medicines and I have been watching my health for 6 years now and I actually confirmed that his herbal medicines are a permanent cure and I'm so happy that I came to know of his herbal healings.You can contact Dr Itua herbal center Email: drituaherbalcenter@gmail.com  WhatsApp: +2348149277967.  if you went through exactly what I go through in terms of health conditions because really honest there is more to learn about natural herbs than medical drugs.